You are on page 1of 63

Din sumar :

* Articole ]i note matematice


* 100 de probleme propuse  pentru clasele I - IV
* 175 de probleme teme  pentru clasele V - XI
* Lucr[ri de verificare  pentru clasele V - XII
* Probleme propuse  pentru clasele V - XII
* Rubrica rezolvitorilor

Anul XXVII , Nr. 2


2016 / 2017
R.M.C.

Sumar
1. ARTICOLE }I NOTE MATEMATICE
 6 SOLUTIONS AND 6 EXTENSIONS FOR AN INEQUALITY - prof. Marin Chirciu
- prof. Daniel Sitaru . . . . . . . . . . . . 1
 TWO SOLUTIONS OF A PROBLEM FROM THE COLLEGE MATHEMATICS JOURNAL
- prof. D. M. B[tine\u-Giurgiu, prof. Neculai Stanciu . . . . . . . . . . . . . . . . . . . . . . . . . . . . . . . . . . . . 8
 NAPOLEON’S THEOREM - prof. Marian Dinc[ . . . . . . . . . . . . . . . . . . . . . . . . . . . . . . . . . . . . . . . 9
 C~TEVA APLICA|II ALE }IRURILOR RECURENTE LA INTEGRALE - prof. Otilia Dr[gan . . . . 10
2. PROBLEME PROPUSE  CICLUL PRIMAR . . . . . . . . . . . . . . . . . . . . . . . . . . . . . . . . . . 13
3. PROBLEME - TEME
 Gimnaziu . . . . . . . . . . . . . . . . . . . . . . . . . . . . . . . . . . . . . . . . . . . . . . . . . . . . . . . . . . . . . . . . . . . . . . . 21

 Liceu . . . . . . . . . . . . . . . . . . . . . . . . . . . . . . . . . . . . . . . . . . . . . . . . . . . . . . . . . . . . . . . . . . . . . . . . . . 29

4. LUCR{RI DE VERIFICARE  Clasele V - XII . . . . . . . . . . . . . . . . . . . . . . . . . . . . . . . . . . 36


5. PROBLEME PROPUSE
 Gimnaziu . . . . . . . . . . . . . . . . . . . . . . . . . . . . . . . . . . . . . . . . . . . . . . . . . . . . . . . . . . . . . . . . . . . . . . . 51

 Liceu . . . . . . . . . . . . . . . . . . . . . . . . . . . . . . . . . . . . . . . . . . . . . . . . . . . . . . . . . . . . . . . . . . . . . . . . . . 54

6. RUBRICA REZOLVITORILOR . . . . . . . . . . . . . . . . . . . . . . . . . . . . . . . . . . . . . . . . . . . . . 57

Redac\ia R.M.C.
Director editor : prof. D[nu\ Dr[cea
Redactor coordonator : prof. Dan Secl[man
Colectivul de redac\ie : @nv. Raluca Dinu Diaconu, @nv. Mariana Ungureanu, prof. Constantin
Basarab, prof. Constantin Cazacu, prof. Dorina Dr[cea, prof. Liliana
Niculescu, prof. Ion P[tra]cu, prof. Lucian Tu\escu, prof. Nicolae T[l[u,
prof. Daniel Al. Ion, prof. dr. Grigorie Preoteasa, prof. Raluca Ciurcea,
prof. Felician Preda, prof. Otilia Dr[gan, prof. Octavian Ungureanu,
prof. C[t[lin Cristea, prof. Cristian Paul Moan\[, prof. C[t[lin Spiridon,
prof. Cerasela Bociu, prof. Delia Mitran.
Colaboratori : prof. Dan Popescu (Suceava), prof. Traian Ianculescu (Zimnicea), prof. Emilia Velcea
(Lupeni), prof. univ. dr. Dumitru Acu (Sibiu), prof. univ. dr. Cristinel Mortici
(T`rgovi]te), prof. univ. dr. Florentin Smarandache (New Mexico, Arizona, U.S.A),
prof. dr. Dorin M[rghidanu (Corabia).
Adresa redac\iei: EDITURA CARDINAL:
Str. R[zboieni, Nr. 2, Bl. B13, Sc. 1, Ap. 2, Craiova, Cod po]tal: 200667
Telefon: 0744.555376; 0351.422424 Fax: 0351.422425


© Editura CARDINAL E Tehnoredactare computerizat[:
All Rights Reserved C Irina Liliana Suciu (Editura Cardinal)
REVISTA DE MATEMATIC{ DIN CRAIOVA 

Publica\ie semestrial[ pentru elevi ]i profesori

 E ANUL XXVII Nr. 2


anul ]colar 2016/2017
C

* ARTICOLE }I NOTE MATEMATICE *


***************************************************************************

 6 SOLUTIONS AND 6 EXTENSIONS


FOR AN INEQUALITY

Prof. MARIN CHIRCIU, Pite]ti


Prof. DANIEL SITARU, Dr. Tr. Severin
Abstract: In this paper are founded several ways to develop a gived inequality .
Let a, b  0. Prove that:
(a  b ) 2
4  2ab  a  b ab 
2 2
2  .
ab 2 2ab a2  b2 ab
Daniel Sitaru, AOPS, 23 Decembrie 2016
Solution 1. By Soumava Chakraborty-Kolkata-India:
(a  b ) 2 (a  b ) 2
 a  b  4ab 2  32 ab 1
4 a
LHS  3  
ab ab (a  b ) 4ab ab 4ab
(a  b ) 2 (a  b ) 2
6 ab  . Suffice it to show that: 6  a  b  52 a  b
ab 4ab ab 4ab b a
(a  b ) 2
2(a 2  b 2 )
which is equivalent to 1  a  b   , (1).
ab 4ab ab
(a  b ) 2
Now, by the GM - HM inequality, ab  2ab , hence a  b  .
ab ab 2ab
(a  b ) 2 3(a  b )
2
It follows that: a  b  1  1 , (2).
ab 4ab 4ab
(1) and (2) show that the problem will be solved if we manage to prove
3(a  b )  4ab 2(a 2  b 2 )
2
 .
4ab ab
This is equivalent to 3a 2  3b 2  10ab  8a 2  8b 2 , or 5a 2  5b 2  1ab  0, which is
simply 5(a  b )  0.
Solution 2. By Kevin Soto Palacios-Huarmey-Peru:
2 ab
We employ the obvious  1 and 4ab 2  1 to obtain
ab (a  b )
3 (a  b )
2
LHS  6  .
4ab
3 (a  b )
2
Suffice it to prove that 6  52 a  b . Now note that
4ab b a
3 (a  b )
2
1  53 ab 2 a  b , because 5  5 a  b .
4ab 2 4 b a b a 2 4 b a
Solution 3. By Soumava Chakraborty-Kolkata-India:
Using the AM - GM inequality, a  b  ab and 2  1 ,
2 ab ab
LHS  ab  ab  a  b ab  1  1  2 ab  a  b ab  2 
2 2ab ab ab 2 2ab ab
(a  b ) 2 a  b (a  b ) 2 2(a  b )
 ab 4  4  . Thus, suffice it to prove that
ab 4ab ab 4ab ab
(a  b ) 2 2(a  b )
4  52 a  b .
4ab ab b a
(a  b ) 2 2(a  b )
 1  2 a  b  2a  2b  ab .
2 2 2 2
Which is equivalent to 
4ab ab ab ab

1  a  b we get 2(a  b )  (a  b ) , implying


2
Now, from
ab 2ab ab ab
(a  b ) 2 2(a  b ) (a  b ) 2 (a  b ) 2 5(a  b ) 2
   a  . Thus, suffice it to prove
4ab ab 4ab 4ab

5 (a  b )  2a 2  2b 2  ab , I.e., 5a 2  5b 2  10ab  8a 2  8b 2  4ab, which reduces


2

4 ab 2 ab
to 3(a  b )  0.

2 Revista de matematic[ din Craiova


Solution 4. By Abdallah El Farissi-Bechar-Algerie:

Let: A  2ab  ab  a  b ab 


1  2 
ab 2 ab ab
2ab

 2ab  ab  a  b 2  1  a  b  1 2ab  ab  a  b 2 
ab 2 ab ab 2ab ab a  b 2

 1 ab  a  b  1 ab  2 ab (a  b )  (a  b )  1 (ab  2(a  b ) ) 
2 2 2
ab ab ab
 1 (5ab  2(a 2  b 2 ))  5  2 a  b .
ab b a
Solution 5. By Soumava Chakraborty-Kolkata-India:
Define  2ab , y  ab , z  a  b . We have x  y  z and y 2  xz.
ab 2
 x (xy  yz  zx )  x (xy  yz  y 2 )
1 
 x cycl

cycl cycl
LHS  x xyz  xyz 
cycl

(x  y  z ) 2 (x  y  z ) 2 2(a  b )  ab
2
. RHS  2a  2b  5ab 
2 2 2
 xz   1  8z2 .
y 2 ab ab y
(x  y  z ) 2 2
The required inequality is equivalent to  1  8z2 . Which is
y 2 y
(x  y  z ) 2  y 2  8z 2 , or, (x  2y  z )(x  z )  8z 2 .
Since x  y  z, x  2y  z  4z and x  z  2z, which shows that, indeed,
(x  2y  z )(x  z )  8z 2 .
Solution 6. By Daniel Sitaru-Romania:
We know that, for positive a, b, c, 2ab  ab  a  b . We’ll use Schweitzer’s
ab 2
n n (m  M ) n2 2
inequality:  x k  x1k  , where x 1 , , x n  m, M, m  0.
k1 k1 4mM
With n  3, m  x 1  2ab , x 2  ab , and x 3  a  b  M, we directly get
ab 2
2ab  a  b 2
A  2ab  ab  a  b a  b  1  2 9 ab
2

ab 2 2ab ab ab 4ab

 2ab  a  b  2ab  a  b
2 2 2
 9 2ab  9
2
ab 
4ab ab 2 4ab 2

 9 3ab  a  b
2
9 14ab  (a 2  b 2 )  63  9 a  b .

4ab 2 16ab 8 16 b a
Now, 1  1 a  b . It then follows that 23  23 a  b and, subsequently,
2 b a 8 16 b a

Editura Cardinal 3
63  9 a  b  40  32 a  b  5  2 a  b .
8 16 b a 8 16 b a b a
This double inequality can be developed.
Let a, b  0. Prove that:
Extension 1: 4  2ab  a  b ab 
2 2
2  4  2n  n a  b ,
ab 2 2ab a2  b2 b a
with n  1 .
2
Proof (Marin Chirciu): We denote
2a  x, a 2  b 2  y and E  2ab  a 2  b 2 ab  2 .
ab 2 ab 2 2ab a2  b2
x2  y2
We have E  (x  y ) 1x  1y  2  xy . We look for an inequality having the
form E  k  n a  b .
b a
2ab 2  a 2  b 2
x y
 2  xy  2  a  b
2 2
We obtain E  k  n a  b 2
2
b a 2ab  a  b 2
2
ab 2
2ab 2  a 2  b 2
( 2ab ) 2  a b
2 2

 ab
2
 2  ab 2
kn a  b , where the penultimate ine-
2ab  a  b ab b a
ab 2
quality follows from: a  b  a  b , and the last is equivalent with:
2 2

2 2
2ab  2ab 2
 a 2
 b 2
 kab  n(a 2  b 2 )  4ab(a  b )  8a 2 b 2 
2
ab 2
(a 2  b 2 )(a  b ) 2  2kab(a  b ) 2  2n(a 2  b 2 )(a  b ) 2  (2n  1 )a 4 
(2k  4n  6 )a 3 b  (4k  4n  18 )a 2 b 2  (2k  4n  6 )ab 3  (2n  1 )b 4  0, (1).
In Horner’s scheme we put the condition that 1 to be double root and we obtain:
8k  16n  32  0, 16k  32n  64  0, where from k  2n  4. It follows that (1) can be
written: (a  b )  (2n  1 )a 2  (2k  8n  8 )ab  (8k  18n  33 )b 2   0 
2

 (a  b ) (2n  1 )a 2  4nab  (2n  1 )b 2   0.


2

Putting the condition that the right parenthesis to be positive we obtain:


2n  1  0.
The equality holds if and only if a  b. Obviously E  (x  y ) 1x  1y  4.
Remark: We denote the arithmetic means, the geometric means, the harmonic
means, respectively the square means with: Ma  a  b , M g  ab , Mh  2ab ,
2 ab
Mp  a 2
 b 2
. The inequality from extension 1 is: 4  (M h  M p ) M  M  1 1
2 h p
a b
 4  2n  n  a , where n  . 1
b 2
We will prove that: 4  x  y ) 1x  1y  4  2n  n a  b
( , where n  1 or
b a 4

4 Revista de matematic[ din Craiova


n  1 , for x, y  M a , M g , M h , M p .
2
Extension 2: 4  a  b  ab 2  1  4  2n  n a  b , with n  1 .
2 ab ab b a 4
Proof (Marin Chirciu):

We denote a  b  x, ab  y and E  a  b  ab 2  1 .
2 2 ab ab
x 2
 y 2
We have E  (x  y ) 1x  1y  2  xy . We look for an inequality having the
a  b 2  ab
x y
2 2
form E  k  n a  b . We obtain E  2  xy  2  2 2
b a a  b  ab
2
a  b 2  ab a  b 2  ab
(a  b ) 2
 2 2 2 3 kn a  b , where the penul-
a  b  2ab ab 4ab b a
2 ab
timate follows from ab  2ab , and the last one is equivalent with:
ab
(a  b ) 2 n (a 2  b 2 )
 (k  3 )   (4n  1 )(a 2  b 2 )  (4k  14 )ab  (4n  1 )a 2 
4ab ab
(4k  14 )ab  (4n  1 )b 2  0, (1). Putting the condition   0 we obtain k  2n  4
and (1) can be written:  (4n  1 )(a 2  b 2 )  (8n  2 )ab  0  (4n  1 )(a  b )  0,
2

true for n  1 .The equality holds if and only if a  b.


4
Extension 3:
4  a  b  2ab 2  a  b  4  2n  n a  b , with n  1 .
2 a  b a  b 2ab b a 4
Proof (Marin Chirciu):
We denote a  b  x, 2ab  y and E  a  b  2ab 2  ab .
2 ab 2 a  b a  b 2
1 1 x2  y2
We have E  (x  y ) x  y  2  xy . We look for an inequality having the
a  b 2  2ab 2

x y 2 2
2 ab
form E  k  n a  b . We obtain E  2  xy  2  2
b a a  b  2ab
2 ab
ab 2
 2ab 2


2 ab kn a  b , where the last one is equivalent with:
ab b a
a  b 2  2ab 2  (k  2 )ab  n(a 2  b 2 )  (a  b ) 4  16a 2 b 2 
2 ab
 (4k  8 )ab(a  b ) 2  4n(a 2  b 2 )(a  b ) 2  (4n  1 )a 4  (4k  8n  12 )a 3 b 
(8k  8n  38 )a 2 b 2  (4k  8n  12 )ab 3  (4n  1 )b 2  0, (1).
In Horner’s scheme we put the condition that 1 to be double root and we obtain:
16k  32n  64  0, 32k  64n  128  0, Where from k  2n  4. It follows that (1) can

Editura Cardinal 5
be written (a  b ) (4n  3 )a 2  (4k  16n  14 )ab  (16k  36n  65 )b 2   0 
2

 (a  b ) 2 (4n  1 )a 2  (2n  2 )ab  (4n  1 )b 2   0.


Putting the condition that the right parenthesis to be positive we obtain 4n  1  0.
The equality holds if and only if a  b.
Extension 4: 4  a  b  a  b
2 2
2  2  4  2n  n a  b ,
2 2 ab a2  b2 b a
with n  1 .
4
Proof (Marin Chirciu):
We denote a 2 b  x, a 2 b  y and E  a 2 b  a 2 b
2 2 2 2
2  2 .
ab a2  b2
x y
2 2
We have  (x  y ) 1x  1y  2  xy . We look for an inequality having the
a  b 2  a2  b2
x 2
 y 2
form E  k  n a  b . We obtain E  2  xy  2  2 2 2
b a a  b  a2  b2
2 2
a  b 2  a2  b2
2(a 2  b 2 )
 2 2 3 kn a  b , where the penultimate
ab  ab (a  b ) 2 b a
2 2
inequality follows from: a  b  a  b , and the last one is equivalent to
2 2

2 2
2 (a  b ) (
2 2
n (a  b )
2 2
 k  3)   na 2  (k  2n  5 )a 3 b  (2k  2n  6 )a 2 b 2 
(a  b ) 2 ab
(k  2n  5 )ab 3 nb 4  0, (1). In Horner’s scheme we put the condition that 1 to be
double root and we obtain: 4k  8n  16  0, 8k  16n  32  0, where from k  2n  4.
It follows that (1) can be written (a  b ) na 2  (k  4n  5 )ab  (4k  9n  16 )b 2  
2

 0  (a  b ) 2 na 2  (2n  1 )ab  nb 2   0, true for n  14 . Putting the condition that


the right parenthesis to be positive we obtain 4n  1  0. The equality holds if and only
if a  b.

Extension 5: 4  ab  2ab 1  a  b  4  2n  n a  b , with n  1


ab ab 2ab b a 4
Proof (Marin Chirciu):

We denote ab  x, 2ab  y and E  ab  2ab 1  ab .


ab ab ab 2ab
x 2
 y 2
We have E  (x  y ) 1x  1y  2  xy . We look for an inequality having the
2

 ab  2ab
ab 2
2 2
x y
form E  k  n a  b . We obtain: E  2  xy  2 
b a ab 2ab
ab

6 Revista de matematic[ din Craiova


ab  2ab 2
ab 3 ab (a  b ) 2 (a  b ) 2
  3   3  kn a  b , where
2ab  2ab 2ab 2 4ab 4ab b a
ab ab ab
the penultimate inequality follows from ab  2ab , and the last one is equivalent
ab
(a  b ) 2 n (a 2  b 2 )
with  (k  3 )   (4n  1 )(a 2  b 2 )  (4k  14 )ab 
4ab ab
(4n  1 )a 2  (4k  14 )ab  (4n  1 )b 2  0, (1). Putting the condition   0 we obtain
k  2n  4 and (1) can be written: (4n  1 )(a 2  b 2 )  (8n  2 )ab  0 
 (4n  1 )(a  b ) 2  0, true for n  14 . The equality holds if and only if a  b.

Extension 6: 4  ab  a2  b2 1  2  4  2n  n a  b ,
2 ab a2  b2 b a
with n  1 .
2
Proof (Marin Chirciu):
We denote ab  x, a 2  b 2  y and E  ab  a2  b2 1  2 .
2 2 ab a2  b2
x2  y2
We have E  (x  y ) 1x  1y  2  xy . We look for an inequality having the
form E  k  n a  b .
b a
ab  a  b ab  a  b
2 2 2 2
x2  y2 2 2
We obtain: E  2  xy  2  2 2
a 2  b2 2ab  a  b
ab  ab 2
2
ab  a  b
2 2

3 a b kn a  b
2 2
 2 , where the penultimate inequality fo-
ab 2ab b a
llows from a  b  a  b and ab  2ab , and the last one is equivalent with:
2 2

2 2 ab
a  b  (k  3 )  n(a  b )  (2n  1 )(a 2  b 2 )  (2k  6 )ab  0 
2 2 2 2

2ab ab
 (2n  1 )a  (2k  6 )ab  (2n  1 )b 2  0, (1).
2

Putting the condition   0 and we obtain k  2n  4 the inequality (1) can be


written (2n  a  b )  0, true for n  1 .
2
The equality holds if and only if a  b.

Editura Cardinal 7
 TWO SOLUTIONS OF A PROBLEM FROM THE
COLLEGE MATHEMATICS JOURNAL

Prof. D. M. B{TINE|U-GIURGIU, Bucure]ti


Prof. NECULAI STANCIU, Buz[u

In CMJ, Vol. 48, No. 1, January 2017, pp. 61-62 was appeared the solution of the
following problem:

1069. Proposed by Angel Plaza, Universidad de Las Palmas de Gran Canaria, Spain.
Let u n  n0 be a sequence defined recursively by u 0  0, u 1  0 and u n1 
 u n  u n1 , for n  1. Determine lim u in terms of u 0 , u 1 .
n n
Here we present other two solutions for the above problem.
Solution 1. If u 1  0, or u 0  0, then u n  0, ( ) n  2, so lim u  0.
n n
If u0 , u1  0, we have u2n2  un1 un , () n  N.
So, u2k2  uk1 uk , () k  N, which yields that
n n n

k0
u 2k2   u k1  u k
k0 k0
 (u 2 u 3 u n u n1 u n2 )  (u 1 u 2 u n u n1 )(u 0 u 1 u n ) 
2

 u 2n2 u 2n1  u 21 u 0 u n1  u 2n2 u n1  u 21 u 0 , (1).


Hence, if we denote u lim n n
u , then by (1) we obtain that
2
lim
n
(u n2 u n1 )  u 1 u 0  u 3  lim
2 2
u
n n
 u 21 u 0  u  3 u 0 u 21 .
Solution 2. If u 1  0, or u 0  0, then u n  0, ( ) n  2, so lim u  0.
n n
If u0 , u1  0, we have un2  u n1 un ,  n  N  ln u 2n2  ln un1 
2 ( )
 ln u n  2 ln u n2  ln u n1  ln u n , (1).
If we denote x n  ln u n , then by (1) yields that 2x n2  x n1  x n  0, which is a
linear recurrence, with the equation 2r 2  r  1  0. We have:

r 1  1, r 2   1 , x n  Ar n1  Br n2  A  (1 ) Bn ,
n
2 2
x  2x 1
and by the initial conditions we obtain x 0  ln u 0 , x 1  ln u 1 , so A  0 ,
3
2(x 0  x 1 ) x  2x 1 ( ) n 1 2(x 0  x 1 )
B , and x n  0  1  . Therefore:
3 3 2n 3
lim
n n
x  1 (x 0  2x 1 )  1 (ln u 0  2 ln u 1 )  1 ln(u 0 u 21 ) lim (ln u n ) 
3 3 3 n

 1 ln(u 0 u 21 )  u  lim u  3 u 0 u 21 .
n n
3

*
8 Revista de matematic[ din Craiova
 NAPOLEON’S THEOREM

Prof. MARIAN DINC{, Bucure]ti

In the paper proof the following theorem use complex numbers.

Theorem:
If similar triangles  PCB   CQA   BAR are erected externally on the sides of
any triangle  ABC, their circumcenters form a triangle similar to the three triangles
 PCB   CQA   BAR such that: QAC  RAB  ; RBA  PBC  ;
PCB  QCA  . Let: O p , O q , O r the circumcenters of the triangles PCB, CQA,
BAR. Let triangle A 1 B 1 C 1 having angles , ,  and circumcenter O 1 . Let x the
complex coordinate of the point X.
B 1 O 1 C 1  2, C 1 O 1 A 1  2, A 1 O 1 B 1  2.
c1  o1 b  op c  o1
 cos 2  i sin 2; BO p C  2 and c  o p  cos 2  i sin 2  1 ,
b1  o1 b1  o1
b  op c1  o1
rezult: c  o p  and b(b 1  o 1 )  c(c 1  o 1 )  o p (b 1  c 1 )  o p 
b1  o1
b(b 1  o 1 )  c(c 1  o 1 )
 .
(b 1  c 1 )
c  oq a o c  oq a  o
AO q C  2, a  o q  cos 2  i sin 2  c 11  o 11 , rezult: a  o q  c 11  o 11 ,
c(c 1  o 1 )  a(a 1  o 1 ) a  or
we obtain: o q  c1  a1 , BO r A  2,  cos 2  i sin 2 
b  or
b o a  or b1  o1 a (a 1  o 1 )  b (b 1  o 1 )
 a 11  o 11 , rezult:  , we obtain: o r 
b  or a1  o1
.
a1  b1
b(b 1  o 1 )  c(c 1  o 1 ) a(a 1  o 1 )  b(b 1  o 1 )
op  or 
b1  c1 a1  b1
oq  or  
c(c 1  o 1 )  a(a 1  o 1 ) a(a 1  o 1 )  b(b 1  o 1 )
c1  a1 
a1  b1
b(b 1  o 1 )(a 1  c 1 )  a(a 1  o 1 )(b 1  c 1 )  c(c 1  o 1 )(a 1  b 1 )
(b 1  c 1 )(a 1  b 1 ) (c  a 1 )
  1 
b(b 1  o 1 )(c 1  a 1 )  a(a 1  o 1 )(c 1  b 1 )  c(c 1  o 1 )(a 1  b 1 ) (b 1  c 1 )
(c 1  a 1 )(a 1  b 1 )
a1  c1
 , rezult:  O p O q O R   A 1 B 1 C 1 .
b1  c1

Editura Cardinal 9
 C~TEVA APLICA|II ALE }IRURILOR
RECURENTE LA INTEGRALE
Prof. OTILIA DR{GAN, Craiova

1
2n
1. Pentru n  N se definesc func\iile f n : R  R, f n (x )  2xn . Fie I n   f n (x )dx.
e x 0
a) S[ se calculeze I 0 .
b) S[ se verifice rela\ia f n1 (x )  1 f n (2x ), ( ) x  R, ( ) n  N.
2
c) S[ se arate c[ I n1  1 I n , ( ) n  N.
4
d) Determina\i termenul general al ]irului S n  I 0  I 1  I 2  . . .  I n ]i calcula\i
limita sa.
1 1 1 1 1
Solu\ie: a) I 0  exx dx   x  e x dx  xe x   e dx  1
e e
x x
 1  2e .
0 0 0 0 0
b) 1 f n (2x )  1  2n1
2x  f (x ).
2 1
2 e x 1 n1 1
2 n1 2 n1 2 n1
c) I n1   f n1 (x )dx   f n1 (x )dx   x  e 2
n1 x
dx.
0 0 0
F[c`nd schimbarea de variabil[ t  2x rezult[ dt  2dx ]i t  0, 1n implic[
1 1
2
2n 2n
 t e 2 n t  1 dx  1
2 2 4
 t dt  1 I , ( ) n  N.
2e 2n t 4 n
0 0

I
d) Din I n1  1 I n , ( ) n  N  n1  1 , ( ) n  N, deci (I n ) n0 este o
4 In 4
progresie geometric[ cu ra\ia 1 ]i primul termen I 0 .
4
|in`nd cont de rela\ia de recuren\[ ob\inem I n1  1 I n , ( ) n  N ]i avem:
2 n1
4 2 n1
1
Sn  I0  I0  1  I0  . . .  1 I0  I0  1  1  1  . . .  1 
4 4 4 4 4 4
n
1 1
 I0 4  4 I  1  1 n , de unde lim S  4 I  4 1  2 .
3 0 4 n n 3 0 3 e
1 1
4
2. Se consider[ func\ia f : 0, 1  R, f (x )  2 1 ]i ]irul (I n ) n N , definit ast-
1 1
x  4x  5
fel I 0  f (x )dx, I n  x n f (x )dx, n  N  .
0 0
a) S[ se calculeze I 0 , I 1 .
b) S[ se demonstreze c[ ( ) n  N, I n2  4I n1  5I n  1 .
n1
c) S[ se arate c[ () n  N, I n1  I n .
d) S[ se demonstreze c[ () n  N , au loc inegalit[\ile 10I n2  1  10I .
n1 n

10 Revista de matematic[ din Craiova


e) S[ se demonstreze c[ () n  N , au loc inegalit[\ile 1  In  1
10(n  1 ) 10(n  1 )
f) S[ se arate c[ lim
n
nI n  f (1 ).
1
Solu\ie: a) x 2  4x  5  (x  2 )  1, deci I 0  1
2
dx.
0 (x  1 ) 2  1
( )
 ( 1) 2 dx    2(x )x 1 dx  arctg(x )  arctg(x  2 ).
x1 1
I 0  arctg(x  2 )  10  arctg3  arctg2.
1 1 1
I 1  x dx  1  x 2 2x dx  1  2x  4  4 dx 
0 x 2  4x  5 2 0 4x  5 2 x 2  4x  5
0

 (xx 2 4x  5)
1 2 1 1
 1 dx  2  x dx  1 ln(x 2  4x  5 )  2I 0 
2 0 4x  5 0 x2  4x  5 2 0

 1 ln 2  2arctg3  2arctg2.
2 1 1 1
b) I n1  4I n1  5I n  x n2 dx  4  2 x dx  5  2 x
n1 n
dx 
0 x 2  4x  5 0 x  4x  5 0 x  4x 5
x n (x 2  4x  5 )
1 1 1 1
 x 2 4x  5x dx   
n2 n1 n
dx  x n
dx  x n dx  1 , n  N  .
0 x  4x  5 0 x 2  4x  5 0 0 n1
x n (x  1 )
1 1 1
c) I n1  I n  x n1 dx  xn dx  dx.
0 x  4x  5
2
0 x  4x  5
2
0 x  4x  5
2

Deoarece x 2  4x  5  0 ]i x n  0, ( ) x  0, 1, rezult[ x  1  0, deci


x  (x  1 )
n
 0, ( ) x  0, 1, adic[:
x 2  4x  5

 xx2 (x4x 1)5 dx  0, deci I n1  I n  0, de unde I n1  I n , ( ) n  N.


1 n

0
d) |in`nd cont de c) are loc I n2  I n1  I n , () n  N adic[ 4I n1  4I n , 5I n  5I n .
Adun`nd membru cu membru se ob\ine: 10I n2  I n2  4I n1  5I n  10I n .
Din b)  10I n2  1  10I n , ( ) n  N  .
n1
e) Din d) avem 10I n  1 , adic[ I n  1 , ( ) n  N  , iar din
n1 10  (n  1 )
10I n2  1 , ( ) n  N  pentru n  2 are loc I n  1 , ( ) n  N  , n  2.
n1 10(n  1 )
1  In  1 , deci n  nI n  n ]i conform criteriului
10(n  1 ) 10(n  1 ) 10(n  1 ) 10(n  1 )
cle]telui rezult[ limn n
I  1  f (1 ).
10 1
3. Se consider[ ]irul de integrale (I n ) n1 , definit prin I n  4x dx.
n

0 x  1
a) S[ se calculeze I 1 ]i I 3 .
b) S[ se demonstreze c[ ]irul (I n ) n1 este descresc[tor.

Editura Cardinal 11
c) S[ se arate c[ I n  I n4  1 .
1
n1
d) S[ se calculeze lim
n
 x n dx.
x41
0
1
e) S[ se calculeze lim n  x n dx .
n
0 x4  1
1 n
f) S[ se demonstreze inegalitatea e  (2n  4 )  xn  e5.
0 x 1
4

1
1 1
(x2 )
Rezolvare: a) I 1   x 4 x 1 dx  12  ( dx  1 arctgx 2  .
x2 )  1 2 8
2
0 0 0
 1
 x 4x 1 dx  14  (xx 4 11)
1 3 1 4
I2  dx  1 ln(x 4  1 )  1 ln 2.
0 4 4 0
4
x  (x  1 )
1 1 1 n
b) I n1  I n  x4 dx  4x dx 
n1 n
dx  0  ]irul (I n ) n1 este des-
0 x  1 0 x  1 0 x4  1
cresc[tor  I n  I n1  2I n  I n  I n4 , dar I n  I n4  1 1
n  1  I n  2n  2 , (1).
I n  I n4  2I n4  I n  I n4 
 1 1
Dar   I n4  2n  2 , ( ) n  N   I n  2n  6 , ( )n 
Dar I n  I n4  1 
n1 
 N , n  5, (2).
Din inegalit[\ile (1) ]i (2)  n  nI  n , ( ) n  N  , n  5 prin trecere
2n  2 n
2n  6
1
n  4x dx  1 .
n
la limit[ ob\inem lim (nI n )  1 lim
2 0 x 1 2
n n

c) Conform punctului precedent avem inegalit[\ile:


1  I  1 , ( ) n  N  , n  5  1  2n  4  (2n  4 )  I  2n  4 , ( )n 
2n  2 n
2n  6 2n  2 n
2n  6
 N, n  5  n  2 n
 ((2n  4 )  I n ) 
n n  2 n
, ( ) n  N  , n  5  prin trecere
n1 n3
n n
la limit[ lim 1 1  lim ((2n  4 )  I n ) n  lim 1 5 
n n1 n n n3 5n
n n3 n3
n1 n1
1 1 1 5
5
lim  lim ((2n  4 )  I n )  lim n

n n1 n n n3
1 n
 e  lim (2n  4 )  x n dx  e5.
n
0 x4  1
Bibliografie:
1 Gh. Andre, Andrei Velicu - “Analiz[ matematic[”, Culegere de probleme.
2 Mircea Olteanu - “Analiz[ matematic[”, No\iuni teoretice ]i probleme.
3 Eugen Popa - “Analiz[ matematic[”, Culegere de probleme.
*
12 Revista de matematic[ din Craiova
************************************************************ *********
* PROBLEME PROPUSE *
*********************************************************************

CICLUL PRIMAR
Clasa I
I.1. Andrada, Alexandra ]i Bianca sunt prietene bune ]i se joac[ cu p[pu]ile.
Andrada are 10 p[pu]i, Alexandra are 9 p[pu]i, iar Bianca are 11 p[pu]i.
C`te p[pu]i au cele trei feti\e?
I.2. Ana ]i Raimonda au plecat @n p[dure s[ culeag[ ciupercu\e. Raimonda a cules
30 de ciupercu\e, iar Ana 20 de ciupercu\e. C`te ciupercu\e au cules cele dou[ feti\e?
I.3. De Cr[ciun, Leonard a primit 11 sold[\ei ]i o ma]inu\[, Gabriel, prietenul lui, a
primit 3 mingi ]i 2 ma]inu\e, iar Ciprian a primit 12 robo\ei.
C`te juc[rii au primit cei trei prieteni buni?
I.4. Se dau cifrele: 1, 2, 9, 4. Forma\i toate numerele de dou[ cifre distincte.
Ordona\i-le cresc[tor!
I.5. La magazin sunt 78 juc[rii: ursule\i, p[pu]i ]i trenule\e. }tiind c[ 61 nu sunt
trenule\e ]i 47 nu sunt ursule\i, c`te sunt din fiecare fel?
I.6. Din succesorul num[rului 48 scade predecesorul num[rului 16.
Afl[ r[sturnatul rezultatului.
I.7. C[su\a ha]urat[ din desenul urm[tor este caracterizat[ de perechea de numere
(2, 3 ) deoarece se afl[ pe al doilea r`nd orizontal ]i pe al treilea r`nd vertical.
Ha]ura\i c[su\ele caracterizate de perechile de numere:
a) (1, 2 ); b) (4, 3 ); c) (3, 4 ); d) (5, 5 ); e) (3, 2 ); f) (1, 4 ).

1 2 3 4 5
1
2
3
4
5

Editura Cardinal 13
I.8. Suma a trei numere naturale diferite este 10. Afla\i numerele ]tiind c[ dou[
dintre ele sunt impare. Scrie\i toate solu\iile.
I.9. Determina\i numerele de dou[ cifre care au suma cifrelor mai mare sau egal[
cu 15 ]i mai mic[ dec`t 17.
I.10. a) Scrie\i numerele de dou[ cifre formate numai din zeci @n care cifra zecilor
nu este mai mic[ dec`t 5.
b) Scrie\i numerele de dou[ cifre formate numai din zeci @n care cifra zecilor
nu este mai mare dec`t 5, nici egal[ cu 5.
I.11. Afla\i numerele de dou[ cifre care au diferen\a dintre cifra zecilor ]i cifra
unit[\ilor: a) 1; b) 2; c) 3.
I.12. Afla\i numerele de dou[ cifre care au diferen\a dintre cifra unit[\ilor ]i cifra
zecilor: a) 4; b) 5; c) 6.
I.13. Scrie\i num[rul 90: a) ca sum[ a trei numere egale formate numai din zeci;
b) ca sum[ a trei numere diferite formate numai din zeci.
I.14. Se dau numere: 8, 13, 5, 22, 45, 21, 56, 99, 53, 78, 14, 48, 62, 37, 4, 9, 61,
80, 75, 87, 94, 36.
a) Scrie\i @n ordine cresc[toare numerele pare.
b) Scrie\i @n ordine descresc[toare numerele impare.
I.15. Completa\i numerele care lipsesc:
a) 13, 15, 17, , 21, , , , , 31, , , , ;

b) 24, 28, 32, , , 44, , , , , , 68, , ;

c) 35, 40, 45, , , , 65, , , , , , , 100.


I.16. Completa\i c[su\ele libere cu semnele “  “ sau “  “:
a) 2 15 2; b) 11 88  100 1;
c) 87 50  22 15; d) 67 15  60 8.
I.17. Verifica\i egalit[\ile ]i completa\i cu A sau F:
a) 4  16  8  10  15  3; b) 23  46  22  68  41  20;
c) 96  45  11  61  33  42; d) 88  24  36  39  14  74.
I.18. Ce num[r trebuie s[ ad[uga\i la suma numerelor 5, 11, 24, 35 pentru a ob\ine
un num[r de dou[ cifre cu cifrele identice? Scrie\i toate solu\iile.
I.19. Efectua\i, grup`nd convenabil, sumele urm[toare:
a) 1  10  89; b) 2  20  78; c) 3  30  67; d) 4  40  56;
e) 5  50  45; f) 6  60  34; g) 7  70  23; h) 8  80  12.
I.20. Efectua\i:
a) 1  11  9  19; b) 2  13  8  10  17; c) 3  15  17  25  13  7.
I.21. Afla\i num[rul necunoscut x, astfel @nc`t suma x  10  x  10 s[ fie egal[ cu:
a) 20; b) 60; c) 100.
Este posibil ca suma s[ fie egal[ cu 30? Dar egal[ cu 31?

14 Revista de matematic[ din Craiova


I.22. Afla\i termenul necunoscut din egalit[\ile urm[toare:
a) a  21  37; b) 17  a  49; c) 88  a  43;
d) a 27  12; e) 25  13  a; f) 44  55  a.
I.23. Care dintre numerele urm[toare este mai mic:
48  24  14; 39  15  14; sau 57  34  13?
I.24. Ionel are 10 bancnote de 1 leu, iar o gum[ de mestecat cost[ 9 lei. Are Ionel
suficien\i bani s[ cumpere o gum[ de mestecat? C`\i lei @i mai r[m`n?
I.25. Pe parcursul unei s[pt[m`ni, 8 elevi din clasa noastr[ ]i-au serbat ziua de
na]tere. Este adev[rat c[ cel pu\in 2 dintre ei ]i-au serbat evenimentul @n aceea]i zi?

Clasa a II-a
II.1. #n vacan\a de iarn[, Mihai a rezolvat 36 de probleme. Dac[ ar mai fi rezolvat
12 probleme, ar avea rezolvate dublul num[rului de probleme rezolvate de fratele s[u.
C`te probleme a rezolvat fratele s[u?
II.2. S[ se completeze c[su\ele libere cu numere identice, la fiecare subpunct:
a)   12; b)    24; c)    36.
II.3. O treime dintr-un num[r este c`t triplul num[rului 3. Care este num[rul
mic]orat de 3 ori?
II.4. #n fiecare din cazurile urm[toare, pune\i @n c[su\ele libere acela]i num[r:
a)  :  8; b)    20;
c)   :  10; d)   :  15.
II.5. Afla\i dublul fiec[ruia dintre numerele urm[toare:
a) 48 : 6; b) 54 : 9; c) 63 : 7; d) 72 : 8 : 3; e) 90 : 9 : 5.
II.6. Afla\i jum[tatea fiec[ruia dintre numerele urm[toare:
a) 4  7; b) 6  9; c) 8  7; d) 2  2  2; e) 2  4  6; f) 3  6  9.
II.7. Afla\i a  b  c ]tiind c[ a, b, c sunt cifre consecutive mai mari dec`t 5.
II.8. Afla\i suma numerelor naturale de dou[ cifre care au diferen\a dintre cifra
zecilor ]i cifra unit[\ilor egal[ cu cifra unit[\ilor.
II.9. Scrie\i num[rul 35 ca sum[ a trei numere naturale, dintre care dou[ s[ fie
egale ]i mai mari dec`t 13. Scrie\i toate solu\iile.
II.10. Afla\i al zecelea termen a ]irului: 1, 1  2, 1  2  3, . . .
II.11. Compara\i numerele: 100  1  12  23  34 ]i 200  41  42  43  44.
II.12. Ordona\i cresc[tor numerele:
123  23  13; 132  32  12; 234  111  12;
213  132  34; 321  231  24; 321  123  88.
II.13. Ordona\i descresc[tor numerele:
199  89  78; 288  79  178; 377  69  278; 466  59  378.

Editura Cardinal 15
II.14. Afla\i numerele de trei cifre cu cifra sutelor 1 care sunt termeni @n ]irul 5, 30,
55, 80, . . .
II.15. Compara\i numerele: 654  321  234  123 ]i 456  123  432  321.
II.16. Separa\i numerele urm[toare @n grupe care au proprietatea c[ toate numerele
dintr-o grup[ au aceea]i cifr[ a sutelor: 135, 432, 234, 423, 324, 132, 243, 342, 246,
357, 420, 157.
II.17. Exist[ numere de dou[ cifre cu proprietatea: x  456  234  123?
II.18. S[ se afle num[rul x ]tiind c[: 344  255  x  255  144.
II.19. Determina\i num[rul care mic]orat cu 55 devine egal cu jum[tatea sa.
II.20. Determina\i num[rul care mic]orat cu 18 devine egal cu treimea sa.
II.21. Diferen\a a dou[ numere naturale este egal[ cu jum[tatea primului num[r.
S[ se afle primul num[r, ]tiind c[ al doilea este 18.
II.22. Produsul a dou[ cifre este un num[r format numai din zeci.
Afla\i cifrele respective. Scrie\i toate solu\iile.
II.23. Produsul a dou[ cifre diferite este un num[r cuprins @ntre 17 ]i 25.
Care sunt aceste cifre? Scrie\i toate solu\iile.
II.24. O clas[ are 36 de elevi. Din ace]tia un sfert au luat nota 10 la lucrarea de
control, o treime din restul elevilor au luat nota 9, o treime din noul rest au luat nota 8,
jum[tate din cei r[ma]i au luat nota 7, iar ceilal\i au luat nota 6.
C`\i elevi au luat nota 6?
II.25. Suma v`rstelor a patru fra\i este un num[r de dou[ cifre format numai din
zeci. Afla\i v`rsta fiec[ruia, ]tiind c[ cel mai mare are 12 ani, iar v`rstele celorlal\i
sunt exprimate prin numere pare diferite. Exist[ mai multe posibilit[\i?

Clasa a III-a
III.1. #n c[mar[ sunt 85 mere: ro]ii, galbene ]i verzi. Dac[ 32 dintre mere nu sunt
nici ro]ii, nici galbene, iar galbene sunt de 4 ori mai pu\ine dec`t verzi, afl[ c`te mere
sunt din fiecare culoare.
III.2. G[se]te pre\ul fiec[rui produs ]tiind c[:
- Un creion reprezint[ o cincime din pre\ul unei c[r\i.
- O carte este cu un leu mai ieftin[ dec`t un stilou.
- Pre\ul stiloului e de trei ori mai mare dec`t al unui caiet.
- 10 lei este de cinci ori mai mare dec`t pre\ul unui caiet.
III.3. Mama cump[r[ pentru copii o p[pu][, o ma]inu\[ ]i o carte. Dac[ p[pu]a
cost[ c`t dou[ ma]inu\e, iar cartea cost[ cu 5 lei mai pu\in dec`t p[pu]a, afl[ c`t a
costat fiecare obiect, ]tiind c[ mama a pl[tit 55 de lei.
III.4. Suma a cinci numere consecutive impare este 145. Afla\i care sunt cele cinci
numere.

16 Revista de matematic[ din Craiova


III.5. Alexandra ]i D[nu\a au @mpreun[ 64 de c[r\i. Alexandra are de 3 ori mai
multe c[r\i dec`t D[nu\a. C`te c[r\i are Alexandra? Dar D[nu\a?
III.6. Maria a economisit bani timp de un an de zile. Cu jum[tate din banii str`n]i a
cump[rat un album de art[, un sfert din banii r[ma]i i-a folosit pentru achizi\ionarea
unor dic\ionare, iar din restul de 45 de lei ]i-a cump[rat o rochie.
Ce sum[ de bani a economisit Maria?
III.7. S[ se determine un num[r de trei cifre @n care cifra sutelor este egal[ cu cifra
zecilor, cifra unit[\ilor este cu 2 mai mic[ dec`t cifra zecilor, iar suma cifrelor este 25.
III.8. Un num[r adunat cu sfertul s[u, cu doimea sa ]i cu 11 d[ 67. Acest num[r
reprezint[ suma v`rstelor a 2 fra\i. Care este produsul v`rstelor lor, ]tiind c[ v`rsta
mezinului este seccesorul lui 11?
III.9. O carte cost[ c`t dou[ penare, iar un penar cost[ c`t patru stilouri. C`t cost[
fiecare obiect, dac[ patru c[r\i, patru penare ]i patru stilouri cost[ 156 lei?
III.10. Pune\i paranteze, unde este nevoie, pentru a ob\ine egalit[\ile:
a) 6  2  5  4  8  3  56; b) 6  2  5  4  8  3  92;
c) 6  2  5  4  8  3  264; d) 6  2  5  4  8  3  540.
III.11. Afla\i a  b  c  d ]tiind c[:
a  b  230; b  c  250; c  d  270; d  a  250.
III.12. }tiind c[ 1000  a  b  b  c  c 1000, afla\i a, b, c.
III.13. Afla\i toate numerele de trei cifre care se scriu sub forma: 123  n  321,
unde n este num[r natural.
III.14. Afla\i dou[ numere naturale consecutive, ]tiind c[ suma lor este:
a) 501; b) 609; c) 839.
III.15. Afla\i trei numere naturale consecutive, ]tiind c[ treimea sumei lor este:
a) 63; b) 156; c) 249.
III.16. G[si\i numerele de trei cifre care au suma cifrelor mai mare dec`t 24 ]i mai
mic[ sau egal[ cu 26.
III.17. Afla\i numerele de patru cifre cu cifrele pare ]i consecutive.
III.18. Afla\i numerele de patru cifre cu cifrele impare ]i consecutive.
III.19. Dac[ adun[m a treia parte dintr-un num[r cu 100, ob\inem jum[tatea acelui
num[r. Determina\i num[rul.
III.20. Determina\i numerele naturale de dou[ cifre care la @mp[r\irea cu 8 dau
restul: a) 7; b) 6; c) 5.
III.21. Determina\i un num[r natural ]tiind c[:
a) m[rind jum[tatea lui de 5 ori, ob\inem num[rul 235;
b) m[rind jum[tatea lui cu 696, ob\inem 931.
III.22. Determina\i dou[ numere naturale ]tiind c[:
a) suma lor este 656 ]i diferen\a 278;
b) suma lor este 475 ]i primul este mai mic de 4 ori dec`t al doilea.
III.23. Suma a cinci numere naturale diferite este 20. Afla\i numerele ]tiind c[

Editura Cardinal 17
exact trei dintre ele sunt pare. C`te solu\ii sunt?
III.24. Afla\i ]ase numere naturale consecutive a c[ror sum[ este 165.
III.25. Alc[tui\i un p[trat folosind 12 dreptunghiuri cu lungimea de 4 cm ]i l[\imea
de 3 cm. Care este latura p[tratului ob\inut?

Clasa a IV-a
IV.1. Un num[r natural este cu 1661 mai mare dec`t altul. Dac[ @mp[r\im num[rul
mai mare la num[rul mai mic, ob\inem c`tul 6 ]i restul 161. Afla\i cele 2 numere.
IV.2. Determina\i numerele de 3 cifre @n care cifra zecilor este egal[ cu cifra
unit[\ilor, iar suma celo 3 cifre este 19.
IV.3. Suma a trei numere naturale este c`t cel mai mare num[r par de 3 cifre. Dac[
din primul num[r sc[dem 50, din al doilea 149, iar din al treilea 196, r[m`n 3 numere
consecutive. Afla\i numerele.
IV.4. a) Afla\i c`te numere naturale de 5 cifre @ncep cu 53.
b) Afla\i c`te numere naturale de 5 cifre se termin[ cu 53.
c) Afla\i c`te numere naturale de 5 cifre distincte @ncep cu 53.
IV.5. Afla\i numerele naturale xyz, astfel @nc`t: xyz  zyx  198.
IV.6. Determina\i numerele naturale de forma abc, ]tiind c[ a  b  10.
IV.7. Suma a trei numere naturale pare consecutive este egal[ cu 30. Afla\i cele trei
numere naturale.
IV.8. Fie ]irul de numere naturale: 12, 41, 70, 99, . . .
a) Scrie\i urm[torii patru termeni ai ]irului.
b) Este num[rul natural 1984 termen al ]irului?
c) Scrie\i al 2016-lea termen al ]irului.
IV.9. Efectua\i calculele urm[toare:
a) 27784 : 46  208  41419  (5078  3065 );
b) 185  216 : 37  58602  (31000  29097 ).
IV.10. Afla\i num[rul necunoscut din urm[toarele egalit[\i:
a) 1997  996  995  (x  5 )  1;
b) 1997  996  95  (x  100 )  1996.
IV.11. Afla\i un num[r natural care este mai mic cu 250000 dec`t triplul s[u.
IV.12. Care dintre numerele 65, 70, 75 trebuie puse @n locul lui x pentru a avea
egalitatea: x  1  2  x  3  x  4  (x  56 )  25.
IV.13. Determina\i num[rul natural cu propriet[\ile:
a) suma dintre o doime a sa ]i o treime a sa este 20100.
b) suma dintre trei doimi ]i dou[ treimi este 46260.
IV.14. }tiind c[ @mp[r\ind pe a la b, ob\inem c`tul 3 ]i restul 300, determina\i
numerele a ]i b dac[ a  b  1700.

18 Revista de matematic[ din Craiova


IV.15. Determina\i suma a cinci numere naturale consecutive ]tiind c[ al treilea
este mai mare cu 1200 dec`t sfertul s[u.
IV.16. Calcula\i a  b  c  d ]tiind c[:
a  b  3300; b  c  5500; c  d  7700; a  d  5500.
IV.17. Afla\i suma numerelor distincte care @mp[r\ite la 9 dau c`tul 123.
IV.18. C`te numere naturale de patru cifre @mp[r\ite la 999 dau restul 99?
IV.19. Afla\i numerele de patru cifre care @mp[r\ite la 1997 dau restul 1996.
IV.20. Determina\i suma tuturor numerelor naturale x pentru care avem inegali-
tatea: 1997  x  1898.
IV.21. Determina\i produsul tuturor numerelor naturale x pentru care avem inegali-
tatea: x  1996  x.
IV.22. Dac[ 15 saci cu f[in[ c`nt[resc cu 675 kg mai pu\in dec`t 24 de saci cu
f[in[ de acela]i fel, c`te transporturi trebuie s[ efectueze un autocamion cu capacitatea
de 4500 kg pentru a transporta 180 de saci cu f[in[?
IV.23. Dac[ m[rim lungimea unui dreptunghi de 3 ori ]i l[\imea de 2 ori, ob\inem
un alt dreptunghi cu perimetrul de 320 m. Afla\i perimetrul dreptunghiului ini\ial,
]tiind c[ l[\imea acestuia este de 20 cm.
IV.24. Anca o @ntreab[ pe bunica sa c`\i ani are. Aceasta @i r[spunde: “C`nd te-ai
n[scut tu, aveam de 4 ori v`rsta ta de acum ]i @nc[ un an, iar @n prezent am mai pu\ini
dec`t 100 cu num[rul care reprezint[ v`rsta ta de acum ]i @nc[ 3 ani.” C`\i ani are
bunica?
IV.25. Bunicul are @n curte p[s[ri ]i animale domestice, @n total 76 de capete ]i 168
de picioare. C`te p[s[ri ]i c`te animale sunt @n curtea bunicului?
Rubric[ realizat[ de @nv. Raluca Dinu Diaconu
Au colaborat la realizarea acestui num[r:
DORINA BR#NDU}E - }c. Gen. Nr. 3, Cugir, DANIELA DR{GOESCU - }c. Gen.
“G. Usc[tescu”, Tg. C[rbune]ti, CAMELIA BULIGIU - }c. Gen. “G. Usc[tescu”, Tg.
C[rbune]ti, MARIANA C#L|ARU - }c. Gen. “G. Usc[tescu”, Tg. C[rbune]ti,
LOREDANA ELENA PETRIC{ - }c. Gen. “G. Usc[tescu”, Tg. C[rbune]ti, DORIN
BR#NDU}E - }c. Gen. Nr. 3, Cugir, NICOLETA CHI|ORAN - Rovinari, Gorj, RODICA
CRAU - }c. Gen. “G. Usc[tescu”, Tg. C[rbune]ti, SILVIA AVRAMESCU - }c. Gen. “G.
Usc[tescu”, Tg. C[rbune]ti, GHERGHINA LUNGEANU - Gala\i, SIMONA-ELIZA
MAZILU - Rovinari, Gorj, ADRIANA C#RSTEA - Tg. C[rbune]ti, LILIANA
ARGENTOIANU - C. N. “Fra\ii Buze]ti”, Craiova, ELENA CIOLACU - }c. Gen.
"Grigore Geam[nu", Turcine]ti, Gorj, GABRIELA MATI} - }c. Gen. Nr. 26, Timi]oara,
DORINA BU}OI - }c. Gen. Bor[scu, Gorj, ANDA TEC}A - }c. cu cls. I-VIII Nr. 3, Cugir,
CORINA ELENA BIANCHI - }c. Gen. “Arcani”, Arcani, ELENA CRI}AN - }c. Gen. Nr.
3 Cugir, DANIELA ANDREESCU - }c. Gen. "Grigore Geam[nu", Turcine]ti, Gorj,
LILIANA VASILESCU - }c. Gen. “G. Usc[tescu”, Tg. C[rbune]ti, ELENA POPESCU -
Lic. cu Prog. Sportiv, Tg. Jiu, ELENA NICOLAE - }c. Gen. Ple]oi, Dolj,
IONELIA-SIMONA NAE - Booveni, Dolj, MARIANA Z{LOG - Tg. Jiu, SIMONA
CROITORU - Gureni, Dolj, IRINA BARBU - Craiova, IOANA PI|A - Vi]ina, Olt,
AUREL }TEFAN - C. N. “Elena Cuza”, Craiova, MARIAN CIUPERCEANU - C. N.
“Fra\ii Buze]ti”, Craiova, LETI|IA DR{GHICI - C. N. “Fra\ii Buze]ti”, Craiova,
VERONICA PRUFU - C. N. “Fra\ii Buze]ti”, Craiova, CRISANDA-GEORGIA TURCU -
C. N. “Fra\ii Buze]ti”, Craiova, IONELA BARB{RAS{ - C. N. “Fra\ii Buze]ti”, Craiova,
BADEA ADELA - Craiova, VILIANA CONSTANTIN - Craiova, LAURA ZAHARIA -

Editura Cardinal 19
Craiova, MIRELA SIMONA POPESCU - C. N. “Elena Cuza”, Craiova, ROXANA
POPESCU - Craiova, LAVINIA VULPE - Craiova, DIANA BORANGIC - Craiova,
LILIANA GENTOIANU - Craiova, EUGENIA PETCU - }c. Gim. "Gh. |i\eica", Craiova,
PETCU VIORICA ANGELICA - }c. Primar[ "Vasile Carabis", C@mpofeni, Gorj, IONICA
TRIC{ - }c. Primar[ P[i]ani, Stoina, Gorj, RODICA CRAU - }c. Gen. "George Usc[tescu",
Tg. C[rbune]ti, ALINA VL{DU|ESCU - }c. Gim. "George Usc[tescu", Tg. C[rbune]ti,
VALENTINA-IRINA COVERC{ - }c. Gim. Ureche]ti, Ureche]ti, Gorj, ANA PAN{ - }c.
Gen., Cru]e\, Gorj, GABRIELA MATI} - }c. Gim. Nr. 26, Timi]oara, ECATERINA
BOGDAN - C. N. "Carol I", Craiova, LICA IONEL OVIDIU - C. N. "Carol I", Craiova,
MARIA DORAN - C. N. "Carol I", Craiova, ION ROTARU - C. N. "Carol I", Craiova,
COCA TANCIU - C. N. "Carol I", Craiova, CATARGENA RADA - C. N. "Carol I",
Craiova, MIHAELA LIC{ - C. N. "Elena Cuza", Craiova, DINU CONSTANTINA - }c. cu
clasele I-VIII, Castranova, CORINA MIRELA ENEA -C. N. "Carol I", Craiova, LIANA
VELI}CU - C. N. "Carol I", Craiova, ISABELLA NICA - C. N. "Carol I", Craiova, IOANA
PI|A - Vi]ina, Olt, ELENA GRIGORE - C. N. "Fra\ii Buze]ti", Craiova, LAURA
ZAHARIA - C. N. “Elena Cuza”, Craiova, ELIZA CHILOM - "}c. cu cls. I-VIII", Murga]i,
SIMONA CECILIA TUDORAN - C. N. "Carol I", Craiova, DENISA BISTRICEANU - C.
N. "Carol I", Craiova, VERONICA COTORA - C. N. "Carol I", Craiova, FLORENTINA
ZULEANU - }c. Nr. 9 "Petrache Poenaru", Craiova, VIRGINIA DOBRESCU - "}c. cu
cls. I-VIII", Desa, IRINA NUT{ - C. N. "}tefan Velovan", Craiova, MIHAELA RISTEA -
C. N. "Carol I", Craiova, IRINA PETRE - C. N. "Carol I", Craiova, GEORGIANA TOMA
- C. N. "Carol I", Craiova, TUDOR C{LIN - "}c. cu cls. I-VIII", D[buleni, ALEXANDRA
C{LIN - Lic. Teoretic "Constantin Br`ncoveanu", D[buleni, ADELA PAULA C{LIN -
Gr[dini\a Nr. 17, Craiova, FLORENTINA POPA - C. N. "Carol I", Craiova, LIANA
VELISCU - C. N. "Carol I", Craiova, GETA TEAU - C. N. "Carol I", Craiova, LIVIANA
G#RD - "}c. cu cls. I-VIII", Ocolna, MIHAELA MICU - "Lic. Teh. Al. Macedonski",
Meline]ti, Dolj, LIDIA EFTENIE - "}c. cu cls. I-VIII", Bor[scu, Gorj, DOINA STOICA ]i
MIRCEA MARIO STOICA - Arad, AURICA ALDEA - "}c. cu cls. I-VIII Nr. 7", Foc]ani,
Vrancea, ELISABETA ALM{}AN - "}c. cu cls. I-VIII Nr. 3", Foc]ani, Vrancea, EMILIA
BATOG - "}c. cu cls. I-VIII Nr. 3", Foc]ani, Vrancea, MARIA COJANU - }c. cu cls. I-VIII,
Nr. 2 "Ion Basgan", Foc]ani, Vrancea, SILVIA CONDREA - }c. cu cls. I-VIII, Nr. 10"Duiliu
Zamfirescu", Foc]ani, Vrancea, PETRIA DOLIA - }c. cu cls. I-VIII, Nr. 10"Duiliu
Zamfirescu", Foc]ani, Vrancea, R{DI|A DR#MBU - }c. cu cls. I-VIII, Nr. 2 "Ion Basgan",
Foc]ani, Vrancea, LUCIA VIORICA GRIGORE - "}c. cu cls. I-VIII Nr. 7", Foc]ani,
Vrancea, MARIANA ICHIM - }c. cu cls. I-VIII, Nr. 2 "Ion Basgan", Foc]ani, Vrancea,
GEORGETA IFRIM - "}c. cu cls. I-VIII Nr. 3", Foc]ani, Vrancea, TITINA MICU - "}c. cu
cls. I-VIII Nr. 3", Foc]ani, Vrancea, ANETA R{DULESCU - Liceul Ped. "Spiru Haret",
Foc]ani, Vrancea, MIRELA RUSU - }c. cu cls. I-VIII, Nr. 5 "Anghel Saligny", Foc]ani,
Vrancea, TUDORI|A STEREA - "}c. cu cls. I-VIII Nr. 3", Foc]ani, Vrancea, LENU}A
STOICA - }c. cu cls. I-VIII, Nr. 2 "Ion Basgan", Foc]ani, Vrancea, FELICIA }TEFAN -
"}c. cu cls. I-VIII Nr. 7", Foc]ani, Vrancea, ZOI|A TABAN - }c. cu cls. I-VIII, Nr. 5
"Anghel Saligny", Foc]ani, Vrancea, GABRIELA T{B{CARU - }c. cu cls. I-VIII, Nr. 8
"Alexandru Vlahu\[", Foc]ani, Vrancea, NELA T{NASE - "}c. cu cls. I-VIII Nr. 3", Foc]ani,
Vrancea, MARI|ICA VI|{ - "}c. cu cls. I-VIII", Soveja, ELENA ZLOTA - }c. cu cls.
I-VIII, Nr. 9 "}tefan cel Mare", Foc]ani, Vrancea, FLORICA DONCA - }c. Gim."Grigore
Moisil", Satu-Mare. LUI|A R{DU| - }c. Nr. 37 "M. Eminescu", Craiova.

*
* *

20 Revista de matematic[ din Craiova


*********************************************************************
* probleme teme *
*********************************************************************

GIMNAZIU

Clasa a V-a
Probleme selectate de: prof. Cristiana Seinu, Craiova

T.G.4645. Ar[ta\i c[ oricare ar fi num[rul natural n, frac\ia 5  24  5 n  5 este


n2 n1 n

2 5
reprezentant al unui num[r natural.
T.G.4646. Ar[ta\i c[ frac\iile urm[toare sunt egale:
a) 3838 ]i 383838 ; b) xy ; xx ]i xxx .
4545 454545 yy yyy
T.G.4647. Ar[ta\i c[ urm[toarele frac\ii sunt ireductibile oricare ar fi n  N.
a) 2n  5 ; b) 7n  10 ; c) 6n  7 .
n2 5n  7 4n  5
T.G.4648. a) Determina\i x  N, astfel @nc`t 4x  5  3.
3x  2 8
2x  5y 1 y
b) Dac[ frac\ia  , calcula\i x .
13x  y 3
c) Dac[ 4a  3b  5 , calcula\i a .
8a  3b 7 b
T.G.4649. S[ se determine n  N ]tiind c[:
2
a) 49  n  400 ; b) 3 n1 3 2 n1 5
64 81 441 5  35  71 ; c) 3  5  7 .
ab  bc  ca aaa  bbb  ccc
T.G.4650. Simplifica\i frac\ia: .
abc  bca  cab aa  bb  cc

T.G.4651. Demonstra\i c[: 49  19n  7  55  N.


n n

2  7  22
T.G.4652. Ar[ta\i c[ num[rul n  1  2  3    70  1  1  1    1 este natu-
2 3 70
ral ]i se divide cu 71.
T.G.4653. Fie frac\ia zecimal[ x  0, 10110111011110. . .
Calcula\i suma primelor 2012 zecimale.

Editura Cardinal 21
T.G.4654. Calcula\i urm[toarea sum[:
S  0, 1  2, 3    6, 7  8, 9  10, 11  12, 13    98, 99  100, 101   
 998, 999    2012, 2013.
T.G.4655. Ar[ta\i c[ num[rul natural x este p[trat perfect, ]tiind c[ verific[ rela\ia:
0, 1  x  0, 3  x  0, 5  x    29, 9  x  9000.
1, (8 )  2, (7 )  3, (6 )    8, (1 )
T.G.4656. Ar[ta\i c[ frac\ia repre-
1, 2 3  2, 3(4 )    6, 7(8 )  7, 8(1 )  8, 1(2 )
( )
zint[ un num[r natural.
T.G.4657. Determina\i cifrele x @n baza 10, care satisfac rela\ia:
1  1  1  N.
x 0, (x ) 0, 0(x )
T.G.4658. Afla\i cifra a, ]tiind c[ num[rul 0, 1(a )  0, (a )  0, a(1 ) este num[r
natural.
1, (1 ) 1, (1 ) 1, (1 )
T.G.4659. Fie S    .
1, 1  2, 2
( ) ( ) 2, 2  3, 3
( ) ( ) 7, 7 )  8, (8 )
(
Ar[ta\i c[ 80  S este divizibil cu 7.
T.G.4660. Determina\i care dintre numerele a ]i b este mai mare:
a 1  1 1  1  1 ; b 1  1 1  1  1 .
2009 2 3 2009 2010 2 3 2010
T.G.4661. Fie mul\imile A  x  N  1  2x  1  11 ;
10 30 45
B  x  N 52x  5x  650; C  x  N x 2  5n  7, n  N.
S[ se calculeze A  C, A  B ]i A  (B  C ).
T.G.4662. #ntr-o zi, la clasele a V-a num[rul elevilor absen\i a fost egal cu 1 din
13
num[rul elevilor prezen\i. A doua zi, num[rul de elevi absen\i a sc[zut cu 1. Astfel,
num[rul elevilor absen\i a fost egal cu 1 din num[rul elevilor prezen\i. Determina\i
20
num[rul total de elevi din clasele a V-a.
T.G.4663. La un concurs elevii ob\in 8 puncte pentru un r[spuns corect, pierd 5
puncte pentru un r[spuns gre]it ]i au 0 puncte dac[ nu au r[spuns. Dac[ au fost 20 de
@ntreb[ri ]i Mihai a ob\inut 13 puncte, la c`te @ntreb[ri a r[spuns corect?
T.G.4664. Cei 24 de elevi ai clasei a V-a au 145 de creioane colorate. #ntr-o grup[,
elevii au c`te 13 creioane fiecare, iar @n alta c`te 5 creioane fiecare ]i @n cealalt[ c`te 4
creioane fiecare. C`\i elevi sunt @n fiecare grup[? G[si\i toate solu\iile.
T.G.4665. a) Un tablou ABCD de tip 3  3 este format din p[trate de tip 1  1.
Fiecare p[trat se poate colora @n 3 culori. #n c`te moduri poate fi colorat tabloul, astfel
@nc`t culoarea s[ fie simetric[ fa\[ de diagonala BD?
b) O alee de 10  10 trebuie pavat[ cu pl[ci p[trate de 1  1, av`nd la
dispozi\ie pl[ci de 4 culori. #n c`te moduri se poate face pavarea, astfel @nc`t aceasta s[
fie simetric[ fa\[ de una dintre diagonale?
T.G.4666. Pe un cerc avem 21 de puncte albe ]i un punct ro]u. Consider[m toate

22 Revista de matematic[ din Craiova


poligoanele determinate de aceste puncte. Care dintre poligoane sunt mai multe: cele
care au toate v`rfurile albe sau cele care au ]i un v`rf ro]u? Cu c`t difer[ num[rul
celor dou[ categorii de poligoane?
T.G.4667. Un cub albastru cu latura de 4 cm se @mparte @n cuburi cu latura de 1 cm.
a) C`te cuburi cu latura de 1 cm vor fi?
b) C`te cuburi mici au exact dou[ fe\e albastre?
c) Dar trei fe\e albastre?
d) Dar nicio fa\[ albastr[?
e) Dar o singur[ fa\[ albastr[?
T.G.4668. Afla\i latura unui p[trat, ]tiind c[ perimetrul p[tratului este cu 4 cm mai
mare dec`t jum[tatea perimetrului unui dreptunghi care are lungimea de 4 m ]i l[\imea
cu 4 cm mai mare dec`t jum[tatea lungimii.
T.G.4669. Un dreptunghi are aria egal[ cu 30 cm 2 . M[rind l[\imea cu 2 cm, aria
noului dreptunghi este egal[ cu 40 cm 2 . Afla\i perimetrul dreptunghiului ini\ial.

Clasa a VI-a
Probleme selectate de: prof. Constantin Basarab, Craiova
T.G.4670. Determina\i mul\imile: A  x  Z  x  1  1  x ]i x  5,
B  x  Z  (2x  1 )  9, C  x  Z  3x  5  Z .
2
2x  3
T.G.4671. Calcula\i: a) (5  7 )  (3  8 )  (6  9 ) : (3  2 );
b) (2 )  (3 )  (1 )  (9 )  3 8 : (27 ) .
4 3 15 5 6

T.G.4672. Un obiect cost[ 60 de lei ]i se ieftine]te cu 20%, iar dup[ un timp se


scumpe]te cu 25%. Afla\i pre\ul final.
T.G.4673. Determina\i numerele @ntregi x, y, z astfel @nc`t 2x  3  2  5
3 3y  1 4z  3
T.G.4674. Rezolva\i @n Z ecua\ia: xy 2  223y 2  224.
T.G.4675. Afla\i x, ]tiind c[ 8a  9b  5c  x(2a  3b  c ).
T.G.4676. Dac[ a, b, c sunt direct propor\ionale cu 3, 4 ]i 2, ar[ta\i c[
a  b  c  30 1  1  1
bc ca ab
.
4  7 7  10 10  13
T.G.4677. Ce condi\ie trebuie s[ @ndeplineasc[ numerele naturale n, p, q pentru ca
num[rul a  (1 )  (1 )  3  (1 )  13. S[ fie divizibil cu 17?
n7 p1 22

T.G.4678. Demonstra\i c[ nu exist[ numere @ntregi x, y, z, astfel @nc`t x 2  y 2  z 2 


x  3y  5z  2017.
3x  y 3y  z 3z  x 3x  y
T.G.4679. Dac[ x, y, z  Z ]i
4x  2y  4y  2z  4z  2x , ar[ta\i c[ 2x  y 
3y  z x
 2y  z  3z
2z  x  4, 5.

Editura Cardinal 23
T.G.4680. Dac[ x ]i y sunt invers propor\ionale cu a ]i b, demonstra\i c[
ab  a  b 2 .
xy xy
T.G.4681. S[ se determine numerele @ntregi a, b, c pentru care a 3 1  b 4 2  c 5 3 .
T.G.4682. Rezolva\i @n numere @ntregi ecua\ia: 2xy  3x  3y  2.
T.G.4683. Calcula\i: 11 12  12  11 11  12  11 10  12  11 9    12  11 2  12  11  12.
T.G.4684. Fie triunghiul dreptunghic ABC cu m A  90  ]i m B  60  . Bisec-
toarea unghiului B intersecteaz[ pe AC @n M. Dac[ MN  BC, N  BC, se cere:
a) m C ; b) MB  MC; c) MN  MA; d) BM  AN.
T.G.4685. Fie triunghiul ABC cu AB  AC. #n semiplanul determinat de BC ]i A
ducem BX  AB ]i CY  AC, iar pe BX ]i CY lu[m punctele M ]i N astfel @nc`t
m BAC
m BMA  m CNA  . Ar[ta\i c[:
2
a) M, A ]i N sunt coliniare;
b) BN  CM;
c) Dac[ BN  CM  P, AP este bisectoarea unghiului BAC.
T.G.4686. Pe laturile AB ]i AC ale triunghiului isoscel ABC, (AB  AC ), consi-
der[m, respectiv punctele D ]i E, astfel @nc`t DE  BC. Fie F un punct pe dreapta DE
situat @n exteriorul triunghiului ABC. Dac[ FG  AC ]i FN  AB(G  AC, H  AC ),
demonstra\i c[ FD este bisectoarea unghiului HFG.
T.G.4687. Se consider[ unghiul XOY ]i pe laturile lui se iau punctele A ]i B, astfel
@nc`t OA  OB. Dac[ D apar\ine bisectoarei unghiului AOB ]i AD  OB, demonstra\i
c[ AD  OB.
T.G.4688. Pe latura BC a triunghiului ABC se ia punctul M. Dac[ N ]i P sunt sime-
tricele lui M fa\[ de AB ]i AC, iar N, A ]i P sunt coliniare, demonstra\i c[ triunghiul
ABC este dreptunghic.
T.G.4689. #n triunghiul ABC cu m B  50  ]i m C  70  , bisectoarea unghiului A
intersecteaz[ @n[l\imea BB   @n M. Ar[ta\i c[ BM  2  MB.
T.G.4690. Fie triunghiul isoscel ABC (AB  AC ) cu m A  36  . #n exteriorul triun-
ghiului ABC se construie]te triunghiul ABE cu AE  EB  BC. Se cere:
a) m AEB ; b) Demonstra\i c[ BE  AC.
T.G.4691. #n triunghiul ABC cu m A  120  , se construiesc @n[l\imile BM ]i
CN, M  AC, N  AB ]i se ia P mijlocul lui BC. Demonstra\i c[ triunghiul MNP
este echilateral.
T.G.4692. Triunghiurile ABC ]i ADC sunt dreptunghice @n B, respectiv D, iar B ]i D
sunt de o parte ]i de alta a dreptei AC. Dac[ M  (BC ), N  (DC ), astfel @nc`t BM 
 DN ]i BAM  DAN, ar[ta\i c[: a) BC  DC; b) AC  BD.
T.G.4693. Fie ABC un triunghi isoscel (AB  AC ) ]i m BAC  36  . Dac[ D este si-

24 Revista de matematic[ din Craiova


metricul lui B fa\[ de C, demonstra\i c[:
a) AC este bisectoarea unghiului BAD;
b) Triunghiul ABD este isoscel.
T.G.4694. Se d[ triunghiul echilateral ABC ]i M  (BC ). Dac[ MP  AC, P  AC ]i
MP  AB  D, demonstra\i c[:
a) MB  BD;
b) AB  2(CP  BN ), unde N este mijlocul lui MD.

Clasa a VII-a
Probleme selectate de: prof. Dorina Dr[cea, Craiova

2 1 3  2 2013  2012
T.G.4695. Calcula\i   .
2 6 2013  2012
T.G.4696. Ar[ta\i c[ num[rul
3 (28  16 3 ) 2014
n  (7  4 3 )   48 este num[r natural.
2013

(7  4 3 ) 2013 2 4030

T.G.4697. Calcula\i a) (2 3  3 2 )  (2 3  1 )  18 ;
2 2

b) 3  2 2  11  6 2  3( 2  1 );

c) 1  1  1  1 .
4  3 5  4 6  5 25  24
T.G.4698. Afla\i media aritmetic[ ]i media geometric[ a numerelor:
a) a  4  2 3 ]i b  4  2 3 ; b) 512 ]i 128; c) 4 5 ]i 16 5 .

T.G.4699. Ar[ta\i c[: 7  9  11    19  14.


12 20 30 90
T.G.4700. Afla\i elementele mul\imii
2 2
M  x  Z x2  a, unde a  5 5 5  5 5 5 .
T.G.4701. Fie mul\imile A   n  n  N , n  564008 ]i B  x  N x  A.
Afla\i cardB.
( 3  2 ) 6 18  45 1
T.G.4702. Calcula\i:   .
( 5  2 ) 7 21  14 3
6 8 10
T.G.4703. Demonstra\i inegalitatea:    2.
8  10 6  10 6  8
2 1 3 1
T.G.4704. Calcula\i:   (3  2 2 )  (4  2 3 )  1 .
2 1 3 1 14

Editura Cardinal 25
2a(2a  1 ) 2a(2a  1 )
2 2
T.G.4705. Calcula\i: a)  ;
3 3
2 2
b) 7  5 : 1 1  16, (3 ) ;
11  2 4  11 3
c) (x  x 2  1 )  (x  2  1 ) .
2 2

T.G.4706. Fie a  2013 4  2  2012 2  4  2013  1 ]i b  2013 4  4  2013  2 


2013 2  3. Ar[ta\i c[ a  b este un num[r natural.
T.G.4707. Rezolva\i @n Z  Z ecua\ia x  y  3x  y  4  0.
T.G.4708. Descompune\i @n factori:
a) a 2  b 2  2a  4b  3; b) (2a  b )  12a  6b  9;
2

c) x 2  25  2x  1; d) x 4  16x 3  64x 2  100.


T.G.4709. Dac[ a  1 1 1
a  8, calcula\i: a  a 2 ; a  a 4 ; a  a 8 .
2 4 8 1

T.G.4710. Fie ABCD un trapez dreptunghic, AB  CD, m A  90  . Dac[ AB 


 Ad  2a, DC  a ]i M este mijlocul AD, ar[ta\i c[ BD  MC.
T.G.4711. Dac[ ABCD este un trapez isoscel cu AB  DC, AB  20 m, DC  10 m ]i
AC  BD, calcula\i:
aria (DOC )
a) aria trapezului; b) perimetrul trapezului; c) .
aria (AOB )
T.G.4712. Dac[ a, b ]i c sunt lungimile laturilor unui triunghi, demonstra\i c[ triun-
ghiul este echilateral, dac[ ]i numai dac[: a  b  c  a  b  c .
2 2 2

abc 3
T.G.4713. Fie unghiul ABC, care nu este obtuzunghic ]i M  (BC ). Demonstra\i c[
dac[ sin BAM  cos CAM , atunci m A  90  .
cos BAM sin CAM
T.G.4714. Se consider[ XOY care are m[sura de 120  . O dreapt[ intersecteaz[
laturile unghiului ]i bisectoarea acestuia @n punctele A, B ]i respectiv C. Ar[ta\i c[:
1  1  1 .
OA OB OC
T.G.4715. Pe cercul C(0, 10 m ) se consider[ punctele A, B, C, D @n aceast[ ordine,
astfel @nc`t m AB  90  , m BC  60  , m CD  120  . Afla\i perimetrul ]i aria pa-
trulaterului ABCD ]i m[surile unghiurilor acestuia.
T.G.4716. Fie triunghiul ABC isoscel cu AB  AC  12 cm, BC  8 cm ]i (AE bisec-
toarea BAC, E  (BC ). Dac[ AE intersecteaz[ cercul circumscris ABC @n punctul
D, calcula\i: a) AD ]i DC; b) Lungimea cercului circumscris;
c) Aria sectorului BOC, unde O este centrul cercului.
T.G.4717. #n  ABC isoscel, cu AB  AC, se consider[ punctele D, E  (AB ), astfel
@nc`t AD  DE  EB. Dac[ CD  10 cm ]i CE  8, 5 cm, calcula\i aria triunghiului
ABC.
T.G.4718. Fie ABCD trapez isoscel, AB  CD, m A  m B  60  . Afla\i aria trape-
zului, dac[ raza cercului @nscris este egal[ cu 12 cm.

26 Revista de matematic[ din Craiova


T.G.4719. Fie triunghiul ABC dreptunghic, m A  90  , AD @n[l\ime, AC  4 3 cm
]i OD  4 cm, unde O este centrul cercului circumscris triunghiului. Calcula\i:
a) Aria perimetrului ABC; b) Lungimea arcului AB;
c) Aria cercului @nscris @n ABC.

Clasa a VIII-a
Probleme selectate de: prof. Constantin Cazacu, Craiova
prof. Lavinia Dalie, Certeju de Sus, Hunedoara
T.G.4720. Se consider[ func\ia f : R R, f (x)  3x  6.
a) Reprezenta\i grafic func\ia.
b) Afla\i coordonatele punctului graficului care are ordonata egal[ cu
triplul abscisei.
T.G.4721. Se d[ f : R  R, f (x )  (1  2 )x  2  1.
a) Ar[ta\i c[ A(1, 0 )  Gf.
b) Rezolva\i @n R inecua\ia f (x )  0.
T.G.4722. Se consider[ func\ia f : R R, f (x)  2x  4.
a) Dac[ Q(a; a  7 )  Gf, determina\i a.
b) Afla\i distan\a de la punctul S(1; 0 ) la graficul func\iei f.
T.G.4723. Se dau func\iile f : R R, f (x)  2x  3 ]i g : R R, g(x)  2x  5.
a) Afla\i coordonatele punctului de intersec\ie al graficelor celor dou[ func\ii.
b) Afla\i aria triunghiului determinat de graficele celor dou[ func\ii ]i axa
ordonatelor.
T.G.4724. Func\ia f : R R, verific[ rela\ia 2 f (x  y)  f (x)  f (y)  x  y  4, pentru
orice x, y numere reale.
a) Reprezenta\i grafic func\ia.
b) Calcula\i suma S  f (1 )  f (2 )  f (3 )  f (4 )    f (39 )  f (40 ).
T.G.4725. Determina\i func\ia f : R R, f (x)  ax  b, ]tiind c[ graficul ei trece prin
punctul A(1; 4 ) ]i f (x )  0, oricare ar fi x  R.
T.G.4726. Se consider[ func\ia f : R R, f (x  2)  5x  2  2 f (1), () x  R.
a) Reprezenta\i grafic func\ia.
b) Afla\i f (x ).
T.G.4727. Ar[ta\i c[ ecua\ia x 2  (a  b ) x  c(a  b  c )  0 are r[d[cini reale,
oricare ar fi a, b, c  R.
T.G.4728. Determina\i func\iile liniare ]tiind c[ graficul formeaz[ cu axa absciselor
un unghi cu m[sura de 30  ]i trece prin punctul A(2; 3 ).
T.G.4729. Rezolva\i @n R ecua\ia 2 1 1 1 x3
x  6x  9  x 2  7x  12  x 2  9x  20  x  5 .

Editura Cardinal 27
T.G.4730. Rezolva\i @n Z  Z ecua\ia 4x2  9y2  4x  5  0.
T.G.4731. Determina\i m  N pentru care ecua\ia (m  1 )x 2  2(m  2 )x  m  2  0
are r[d[cini reale ]i diferite.
T.G.4732. Rezolva\i @n R: a) x  2  4  x  x;
 x  1  y  5  12
b) x  4x  5  0;
2
c)  .
5  2x  x  1  y  13
T.G.4733. O carte cost[ 11 lei ]i un stilou cost[ 7 lei. Care este cel mai mare num[r
de c[r\i ]i stilouri ce se pot cump[ra cu 770 de lei? (f[r[ s[ r[m`n[ rest).
T.G.4734. Dup[ trei ieftiniri succesive cu 10% un produs cost[ 729 lei. Care a fost
pre\ul ini\ial al produsului?
T.G.4735. Dac[ ABCD A  B  C  D  este prism[ patrulater[ regulat[ dreapt[ cu baza
ABCD, AB  8 cm ]i d(D  , (A  BD ))  4 cm, calcula\i:
a) Aria total[ ]i volumul prismei.
b) Sinusul unghiului dintre A  D ]i BC.
c) M[sura unghiului dintre planele (A  BD ) ]i (C  BD ).
T.G.4736. #n cubul ABCD A  B  C  D  punctul M este mijlocul muchiei CC   ]i
A AD  M  300 m 2 . Calcula\i:
a) Aria total[, volumul ]i diagonala cubului.
b) Distan\a de la M la planul (D  BD ).
T.G.4737. #n prisma triunghiular[ regulat[ dreapt[ ABC A  B  C  cu baza ABC, AB 
 8 cm ]i A  B  BC  . a) Ar[ta\i c[ AA   4 2 cm.
b) Calcula\i aria total[ ]i volumul prismei.
T.G.4738. #n paralelipipedul dreptunghic ABCD A  B  C  D  , AB  8 cm, BC  6 cm, M
este mijlocul muchiei AA  ]i d(B , (MBC))  2 2 cm.
a) Calcula\i aria total[ ]i volumul.
b) Calcula\i m(((MBC ), (MAD ))).
T.G.4739. Un vas @n form[ de piramid[ triunghiular[ regulat[ cu @n[l\imea de 8 dm
se umple cu ap[ p`n[ la @n[l\imea de 4 dm ]i apoi se @ntoarce cu v`rful @n jos ]i astfel
apa se ridic[ p`n[ la @n[l\imea h. Afla\i h 3 .
T.G.4740. #n piramida patrulater[ regulat[ VABCD cu baza p[tratul ABCD,
punctele M ]i N sunt mijloacele muchiilor VA, respectiv VD, @n[l\imea VO inter-
secteaz[ planul (MNB ) @n punctul P, iar AB  8 3 cm ]i VB  4 15 cm.
a) Afla\i aria lateral[ ]i volumul piramidei.
b) Afla\i distan\a de la punctul P la MN.
T.G.4741. Se consider[ piramid[ patrulater[ regulat[ VABCD cu latura bazei egal[
cu 12 cm ]i muchia lateral[ egal[ cu 12 2 cm.
Dac[ M  VA, astfel @nc`t m(((VDB ), (ADC )))  30  , calcula\i:
a) Aria lateral[ ]i volumul piramidei.
b) Distan\a de la M la planul (VBC ).

28 Revista de matematic[ din Craiova


T.G.4742. Trunchiul de piramid[ triunghiular[ regulat[ ABC A  B  C  are baza mare
 ABC, AB  12 cm, A  B   3 cm ]i muchia lateral[ AA  formeaz[ cu planul ABC un
unghi cu m[sura de 30  .
a) Calcula\i volumul piramidei din care provine trunchiul.
b) Calcula\i aria lateral[ ]i volumul trunchiului.
c) Calcula\i d(A  , (BCC  )).
T.G.4743. Un con circular drept are desf[]urarea lateral[ un semidisc ]i @n[l\imea
conului este egal[ cu 10 3 cm.
a) Calcula\i aria lateral[, aria total[ ]i volumul conului.
b) Se sec\ioneaz[ conul cu un plan paralel cu baza. Afla\i volumul trun-
chiului de con format, ]tiind c[ lungimea cercului de sec\iune este egal[ cu 8 cm.
T.G.4744. #ntr-un con circular drept cu @n[l\imea egal[ cu 15 cm ]i raza egal[ cu 20
cm se @nscrie o sfer[. Afla\i aria ]i volumul sferei.

LICEU

Clasa a IX-a
Probleme selectate de: prof. Otilia Dr[gan , Craiova

T.L.4025. Se consider[ func\ia f : R R, f (x)  x2  ax  a. Determina\i valorile


parametrului real a astfel @nc`t mul\imea A  x  R  f (x)  0 s[ fie format[ din-
tr-un singur element.
T.L.4026. S[ se determine parametrul real m, astfel @nc`t ecua\ia x 2  (3m  2 )x 
1  5m  0, s[ aib[ r[d[cinile x1 , x 2  R cu proprietatea c[ x 1  x 2  0.
T.L.4027. Fie func\ia f : R R, f (x)  3(m  1)x2  6mx  3m  2, m  1.
S[ se determine m  R, pentru care v`rful parabolei asociate func\iei se afl[ pe dreapta
de ecua\ie y  x  4.
T.L.4028. Fie familia de func\ii f : R R, f m(x)  x 2  2(m  1)x  m  3. S[ se deter-
mine locul geometric al v`rfurilor parabolelor, asociate func\iilor.
T.L.4029. S[ se determine coordonatele punctelor de intersec\ie dintre parabola de
ecua\ie y  x 2  10x  16 ]i dreapta de ecua\ie y  x  2.
T.L.4030. Fie func\ia f : R  R, f (x )  3ax 3  (2a  7 )x  c, a  0. S[ se determine
a, c  R ]tiind c[ func\ia admite un punct de extreme @n 2 ]i care are valoarea 2.
T.L.4031. S[ se determine imaginea func\iei f : R  R, f (x )  x 2  2x  1 .
2

x  2x  2
T.L.4032. Se consider[ ecua\ia: (m  3 )x 2  2(m  2 )x  m  1  0. S[ se determine
m  R, ]tiind c[ @ntre r[d[cinile ecua\iei exist[ rela\ia: 2(x 1  x 2 )  x 1 x 2  3x.

Editura Cardinal 29
T.L.4033. Fie func\ia f : R R, f (x)  2x  1. S[ se determine func\ia g : R R,
]tiind c[ (g  f )(x)  4x  6, () x  R.
T.L.4034. Fie func\ia f : R R, f (x)  3x  1. S[ se calculeze: f  f    f .
2013 ori

T.L.4035. Se consider[ func\ia strict cresc[toare f : R R, f (x)  (3m  2)x  m  4.


S[ se determine m  R, astfel @nc`t graficul func\iei s[ con\in[ punctul A(m  4, 14 ).
T.L.4036. Fie func\ia f : R R, f (x)  3x  4. S[ se determine func\ia g : R R,
]tiind c[ (f  g)(x )  3x2  6x  1, () x  R.
T.L.4037. S[ se determine imaginea intervalului 2, 5 prin func\ia
f : R R, f (x)  x2  7x  10.
T.L.4038. S[ se determine func\ia de gradul I care verific[ rela\ia:
(f  f )(x )  2f (x )  4.
T.L.4039. Fie x 1 , x 2 solu\iile ecua\iei: x 2  3x  5  0.
x2  1 x2  1
S[ se calculeze: 1x 2  2x 1 .
T.L.4040. S[ se calculeze S  sin 7   sin 47   sin 87     sin 327 .
T.L.4041. S[ se determine perioada principal[ a func\iei f : R  1, 1, f (x)  sin 5x
T.L.4042. }tiind c[ tg a  2, s[ se calculeze cos 2a.
2
T.L.4043. Determina\i a  R, pentru care este adev[rat[ egalitatea:
sin 6x  sin 2x  2a  ctg2x  tg4x.
sin 6x  sin 2x
T.L.4044. }tiind c[ sin x  2 cos x  3 , ( ) x  R, calcula\i sin 2x.
T.L.4045. Se consider[ f : R R, f (x)  (x  2)  (x  1), de unde  reprezint[
partea frac\ionar[.
a) S[ se arate c[ f este periodic[ T  1.
b) Explicita\i func\ia.
T.L.4046. Demonstra\i egalitatea: sin 24  sin 2 72 2  sin 36  sin 2 78   cos 12  cos 18  .
3 cos 8   3 sin 8 
T.L.4047. Demonstra\i c[  tg52  .
3 cos 8   3 sin 8 
T.L.4048. Demonstra\i c[ @n orice triunghi ABC este adev[rat[ egalitatea:
tg A  B
ab  2 .
a  b tg A  B
2
T.L.4049. Fie ABC ]i M, N, P  mijloacele laturilor BC, CA, respectiv AB.
          
Demonstra\i c[ AM  BC  BN  CA  CP  AB  0.

30 Revista de matematic[ din Craiova


Clasa a X-a
Probleme selectate de: prof. C[t[lin Cristea, Craiova

T.L.4050. a) S[ se calculeze num[rul de func\ii f care se pot defini pe mul\imea


1, 2 cu valori @n mul\imea 3, 4, 5.
b) S[ se calculeze num[rul de func\ii injective f care se pot defini pe
mul\imea 1, 2 cu valori @n mul\imea 3, 4, 5.
c) S[ se calculeze num[rul de func\ii strict cresc[toare care se pot defini
pe mul\imea 1, 2 cu valori @n mul\imea 3, 4, 5.
T.L.4051. S[ se calculeze num[rul de func\ii bijective ]i care nu sunt strict
monotone, care se pot defini pe mul\imea 1, 2, 3 cu valori @n mul\imea 4, 5, 6.
T.L.4052. S[ se calculeze:
a) Num[rul func\iilor: f : 1, 3, 5  2, 4, 6 pentru care f (1 ) divide f (3 ).
b) Num[rul func\iilor: f : 1, 3, 5  2, 4, 6 care sunt monoton descresc[toare.
T.L.4053. Dac[ M  x  R 2 2x 6x4  2 x 3x3  1  0, s[ se calculeze num[rul
2 2

tuturor func\iilor inversabile definite pe M cu valori @n M.


T.L.4054. a) S[ se determine c`te numere naturale divizibile cu 5, de trei cifre
distincte se pot forma cu elemente din mul\imea 0, 1, 3, 4, 5.
b) S[ se determine c`te numere impare, de cinci cifre distincte se pot
forma cu cifre din mul\imea 1, 2, 3, 4, 5.
T.L.4055. S[ se determine probabilitatea ca o func\ie f definit[ pe mul\imea 1, 3,
5, , 2007 cu valori @n mul\imea 0, 2, 4, , 2008 s[ fie injectiv[.
T.L.4056. a) S[ se determine probabilitatea ca un element n al mul\imii 1, 2, 3,
4, 5 s[ verifice rela\ia n!  n n .
b) S[ se determine probabilitatea ca un element al mul\imii 0, 1, 2, 3, 4
s[ fie solu\ie a ecua\iei 3 x  1  3 x  1 .
T.L.4057. a) Calcula\i: 1  2  3 .
2! 3! 4!
b) Afla\i n  N, astfel @nc`t 1  2  3    n  1  719 .
2! 3! 4! n! 720
T.L.4058. Calcula\i: a) C 17  C 37  C 57 ; b) (C57  C27 ) ;
2007

c) A 4  A 5  C 4  C 5 ;
2 3 2 3
d) C 03  C 13  C 23  C 33 .
T.L.4059. S[ se rezolve:
(n  2007 )!
a) ecua\ia 8  A 5n1  3  P 3  A 5n ; b) inecua\ia  n  2006.
(n  2005 )!
x  1  C y , ( )x, y  N, x  y.
T.L.4060. a) S[ se verifice egalitatea: C y1 x1 
y1 x

b) S[ se rezolve ecua\ia: 2y  C y 2  (y  1 )  C y 2 1 .
y y1

C1 C2 C n (2 n  1 ) 2
T.L.4061. Dac[ n  N , s[ se arate c[: n  n    nn  .
1 2 n  2 n1

Editura Cardinal 31
T.L.4062. S[ se demonstreze c[:
(n  1 ) 2  (2n  1 )
3n1  C 3n  C 3n1  (C 3n1 ) , ( ) n  N, n  1.
n1 3
 C n1 n n1
9n  (2n  1 )  (3n  1 )
k  1 )!
b)  k  k  3k .
n ( n 3 2
T.L.4063. S[ se calculeze sumele: a)  ;
k1 (k  1 )! k1 (k  1 )!
T.L.4064. #n c`te moduri pot fi ordonate orele de matematic[, chimie, istorie ]i
fizic[ @ntre orele 8 00  12 00 , @ntr-o zi a s[pt[m`nii?
T.L.4065. Afla\i termenul independent de x, x  0 din dezvoltarea:
18
1  42 .
1
2  x 5 x
n

T.L.4066. #n dezvoltarea 9x  1 , x  0, suma coeficien\ilor binomiali ai pri-


3x
milor trei termeni este 121. S[ se g[seasc[ termenul care-l con\ine pe x 3 .
1 2 3  n
T.L.4067. Determina\i valoarea medie a variabilei aleatoare X 1 1 1 1 .
n n n  n
T.L.4068. Fie A, B  , P(A )  1 , P B  3 , P A (B )  1 . Preciza\i dac[ evenimen-
3 4 4
tele A ]i B sunt independente ]i calcula\i P(A  B ).
T.L.4069. Un elev @]i cump[r[ un computer, @n urma reducerii de 15% pl[tind 1700
lei. Care a fost pre\ul ini\ial al computerului?
T.L.4070. Se consider[ dou[ v`rfuri consecutive M(2, 1 ) ]i N(4, 3 ) ale paralelogra-
mului MNPQ, precum ]i punctul de intersec\ie al diagonalelor D(3, 5 ). S[ se calculeze
aria triunghiului DPQ.
T.L.4071. Se consider[ punctele A(2, 2 ), B(4, 5 ) ]i dreapta d : x  8  0.
a) S[ se determine coordonatele punctului C  d, astfel @nc`t triunghiulABC s[
fie isoscel cu baza (AB ).
b) S[ se scrie ecua\ia medianei din B a triunghiului ABC.
T.L.4072. Se consider[ sistemul cartezian de coordonate xOy ]i punctele A(3, 0 ),
B(2, 1 ), C(0, 2 ), respectiv D(6, 3 ). S[ se demonstreze c[ dreptele AB, CD ]i perpen-
diculara din O pe AC sunt concurente.
T.L.4073. S[ se determine ,   R, astfel @nc`t vectorii
 (     
u  3  2 ) i  3 j ]i v  (5  4 ) i  3 j s[ fie:
a) egali; b) coliniari.
     
T.L.4074. Se consider[ vectorii v 1  2 i  3 j ]i v 2  6 i  4 j .

a) S[ se calculeze modulul vectorului v 1 .
 
b) S[ se arate c[ vectorii v 1 ]i v 2 sunt ortogonali.

32 Revista de matematic[ din Craiova


Clasa a XI-a
Probleme selectate de: prof. C[t[lin Cristea, Craiova

T.L.4075. Rezolva\i sistemele:


 2x  y  4z  2  4x  2y  3z  7  11
 
a)  5x  4y  z  19 . b)  x  y  z  5t  6 .
 x  y  7z  7  2x  3y  4z  2
 
 ax  y  z  x

T.L.4076. Fie sistemul  x  ay  z  y , a  R.
 x  y  az  z

a) Ar[ta\i c[ determinantul matricei sistemului este egal cu (a  1)(a  2 ) .
2

b) Ar[ta\i c[ sistemul este compatibil pentru orice a  R.


c) Rezolva\i sistemul pentru a  2017.
x
1 1 1 1
y
T.L.4077. Fie matricele A  M 3,4 (R ), A  0 1 1 1 , X  M 4,1 , X  ,B 
z
0 0 1 1
t
1
 M 3,1 (R ), B  0 .
0
a) Determina\i rangul matricei A.
b) Rezolva\i ecua\ia A  X  B.
c) Determina\i mul\imea M  Y  M 4,3 (C )  det(Y  A )  0.
 x  y  (3  2m )z  1
 (
T.L.4078. Fie sistemul:  2m  3 )x  y  (2m  3 )z  4  2m, m  R .
 (2m  3 )x  3y  3z  1

a) Rezolva\i sistemul pentru m  0.
b) Determina\i m pentru care sistemul este incompatibil.
c) Determina\i m pentru care sistemul este compatibil nedeterminat.
 x  2y  z  t  0

 2x  y  3z  3t  0
T.L.4079. Determina\i m pentru care sistemul 
 xyzt 0

 2x  (m  1 )y  2z  mt  0
are solu\ii nenule.
arcsin x
1  x2 tg(2015x )  sin(2016x )
T.L.4080. Calcula\i: a) lim ; b) lim .
x0 ln(1  x ) x0 tg(2017x )  sin(2018x )

Editura Cardinal 33
ln(e 2  x )  ln(e 2  2x )
T.L.4081. a) Calcula\i lim
x0 x .
b) Determina\i valorile lui a  R 1 astfel @nc`t
x x
x1  x
lim
x
1  e2.
(a  1 ) 2 x 2  1
T.L.4082. Se d[ func\ia f : R  R, f (x )  1  x  x 2  x.
a) Calcula\i f  (1 ).
b) Scrie\i ecua\ia tangentei la graficul func\iei f @n punctul de abscis[ x 0  1.
c) Determina\i asimptotele func\iei f.
T.L.4083. Ar[ta\i c[, pentru orice x  2 are loc inegalitatea:
(x  1 ) cos   x cos x  1.
x1
T.L.4084. Fie func\ia f : RR, f (x)  x3  3x  2.
a) Studia\i derivabilitatea func\iei g : R 2, 1  R, g(x )  3 1 .
f (x )
b) Determina\i punctele de pe graficul func\iei f @n care tangenta la grafic este
paralel[ cu dreapta de ecua\ie y  3x.
lim 3 x 3  3x  2  x 2  3x  2 .
c) Calcula\i x
T.L.4085. Fie a1 , a2 , . . ., a2017  R, a1  a2  . . .  a2017  0. Calcula\i:
lim
x
a 1 x  a 1  a 2 x  a 2    a 2017 x  a 2017 .
T.L.4086. Se consider[ func\ia f : (1, ) R, definit[ prin f (x )  ln(x 2  1 )  2 ln x.
a) Calcula\i f  (2 ).
b) Calcula\i limn
f (2 )  f (3 )    f (n )  ln 2 n .
c) Demonstra\i c[ func\ia f este cresc[toare pe (1,  ).
T.L.4087. Se consider[ func\ia f : R  R, f (x )  1  x 2  ax, a  R.
a) Determina\i a  R, ]tiind c[ f  (1 )  0.
f (x )  1
b) Calcula\i lim x .
x0
c) Determina\i a  0 astfel @nc`t dreapta y  0 s[ fie asimptot[ orizontal[ spre  .
T.L.4088. Fie func\ia f : R R, f (x)  arctg(x  2017)  arctgx.
a) Calcula\i f  (x), x  R.
b) Determina\i asimptota spre   la graficul func\iei f.
c) Studia\i monotonia func\iei f.

T.L.4089. Fie func\ia f : R  R, f (x )  x 2 x  1 .


2

x 1
a) Calcula\i f  (x), x  R.
b) Calcula\i lim
x
f (x ) 2017x .
c) Demonstra\i c[ func\ia f nu este surjectiv[.
T.L.4090. Fie func\ia f : R  R, f (x )  e x  e 1x .
2 2

a) Calcula\i f  (0 ).

34 Revista de matematic[ din Craiova


b) Scrie\i ecua\ia tangentei la graficul func\iei f @n punctul de abscis[ x 0  0.
c) Demonstra\i c[ 2 e  f (x )  1  e, ( )x  0, 1.

T.L.4091. a) Determina\i a, b  R, astfel @nc`t: lim 3


ax 3  bx 2  2x   1 .
x 3
b) Determina\i a, b  R, astfel @nc`t: lim
x
x 2  x  1  ax  b  0.
x 2  7x  3arctgx
T.L.4092. Determina\i a  R, astfel @nc`t lim x xa exist[, este finit[ ]i
nenul[.
T.L.4093. Calcula\i lim x ln x ]i lim x x .
x0 x0
T.L.4094. Se consider[ func\ia
 m 2 x  4, x  0
f : R  R, f (x )   ( 4 .
 ln x  1 )  2marctgx  n , x  0
2

a) Demonstra\i c[ func\ia f este continu[ @n x  0, () m  R ]i n  R cu n  2.


b) Determina\i elementele mul\imii B  (m, n)  R  R f este derivabil[ @n x  0.
T.L.4095. Se consider[ func\ia f : R R, f (x)  x2  ex .
a) Calcula\i f  (x), x  R.
b) Determina\i intervalele de convexitate ale func\iei f.
c) Demonstra\i c[ 0  f (x )  f (x 2 )  e e 1 , ( ) x  1, 0.
2

T.L.4096. Calcula\i:
a) I 1  lim sin x  sin 2x  sin 3x  sin 4xx    sin 2016  sin 2017x .
x0

b) I 2  lim 1  cos x  cos 2x    cos 2017x .


x0 x2
T.L.4097. Determina\i parametri reali a ]i b, astfel @nc`t
lim
x
3
x 3  x 2  2x  3  x 2  x  4  ax  b  2.
T.L.4098. Se consider[ func\ia f : R R, f (x)  2x  arctgx.
a) Ar[ta\i c[ func\ia f este bijectiv[.
f (x )  f (1 )
b) Calcula\i lim .
x1 x1
c) Determina\i asimptota c[tre   la graficul func\iei f.
T.L.4099. Se consider[ func\ia f : 0,   R, f (x )  ln(1  x )  x  1 x 2 .
2
a) Determina\i punctele de extrem local ale func\iei f.
b) Ar[ta\i c[ graficul func\iei f nu are asimptote.
c) Demonstra\i c[ x  1 x 2  ln(x  1 )  x, ( ) x  0.
2

*
* *

Editura Cardinal 35
*********************************************************************
* LUCR{RI DE VERIFICARE *
*********************************************************************

CLASELE V-XII

Clasa a V-a
Lucrarea 1 - (timp de lucru: 50 min.) - Se acord[ 10 puncte din oficiu.
Alc[tuit[ de prof. CRISTIANA SEINU, Craiova

SUBIECTUL I. (30 puncte) - Pe foaia de tez[ se trec doar rezultatele.

5p 1. Rezultatul calculului 2, 43  1, 7 este egal cu . . . . . .


5p 2. Cifra sutimilor num[rului 0, 1578 este egal cu . . . . . .
5p 3. Efectu`nd calculul 5, 4  7 ob\inem frac\ia zecimal[ . . . . . .
4
5p 4. Valoarea lui x din ecua\ia x  2, 25  4, 75 este egal[ cu . . . . . .
5p 5. Cel mai mic num[r natural care verific[ rela\ia 4x  17 este . . . . . .
5p 6. Dintre numerele 2, (34 ) ]i 2, 3(4 ) mai mare este num[rul . . . . . .

SUBIECTUL II. (30 puncte) - Pe foaia de tez[ se trec doar rezultatele.


5p 1. #ntr-o lad[ sunt 23, 4 kg de mere. C`te kilograme de mere sunt @n 9 astfel de
l[zi?
5p 2. Calcula\i perimetrul dreptunghiului care are lungimea de 5 cm ]i l[\imea de 3
cm.
5p 3. Calcula\i lungimea laturii unui p[trat care are perimetrul egal cu 28 dm.
5p 4. Calcula\i ]i exprima\i rezultatul @n metri: 5, 23 dam  23 cm.
5p 5. Un autoturism consum[ 0, 016 litri de benzin[ pe o distan\[ de 200 m.
Calcula\i consumul la 100 km.
5p 6. Desena\i axele de simetrie ale p[tratului din figura
al[turat[.

36 Revista de matematic[ din Craiova


SUBIECTUL III. (30 puncte) - Pe foaia de tez[ se trec doar rezultatele.

5p 1. Rezolva\i ecua\ia: x  1  4, 51  22, 5 : 1, 5  0, 4  (33, 6 : 3  2  4, 2 2 ), x 


4
 N.
5p 2. Media aritmetic[ a dou[ numere este 14, 32, iar unul din numere este cu 2, 5
mai mare dec`t cel[lalt.
a) Calcula\i suma celor dou[ numere.
b) Afla\i cele dou[ numere.
10p 3. a) Semiperimetrul unui dreptunghi este egal cu 40 cm. }tiind c[ l[\imea
dreptunghiului este de 3 ori mai mic[ dec`t lungimea sa, afla\i aria dreptunghiului.
b) Afla\i aria unui p[trat care are latura egal[ cu 2 din lungimea dreptun-
5
ghiului.
10p 4. Determina\i valorile naturale nenule ale lui x care verific[ egalitatea:
a, (b )  b, (c )  c, (a )
 1 1x .
a, bc  b, ca  c, ab

Lucrarea 2 - (timp de lucru: 50 min.) - Se acord[ 10 puncte din oficiu.


Alc[tuit[ de prof. CRISTIANA SEINU, Craiova

SUBIECTUL I. (60 puncte) - Pe foaia de tez[ se trec doar rezultatele.


10p 1. Calcula\i 0, 34  1, 2  5  1, 206  . . . . . .
10p 2. Rezolva\i ecua\ia: 3x  42, 3  45, 6.
10p 3. Efectua\i: 3  0, 5  2  (6, 25 : 0, 25  4 ) : 3  . . . . . .
10p 4. Afla\i media aritmetic[ a numerelor: 3, 9; 5, 04 ]i 3, 06.
10p 5. Efectua\i: 40 m  0, 5 hm  300 cm  . . . . . .dm.
10p 6. Compara\i frac\iile zecimale: 11, 12 ]i 11, (2 ).

SUBIECTUL II. (30 puncte) - Pe foaia de tez[ se trec doar rezultatele.


10p 1. a) Desena\i un p[trat MNPQ.
b) Afla\i perimetrul p[tratului ]tiind c[ aria sa este egal[ cu 81 m 2 .
c) Afla\i c`\i litri de ap[ con\ine un cub care are latura egal[ cu latura p[tra-
tului MNPQ.
d) Efectua\i transform[rile: 0, 8105 dm 2  . . . . . . ari  . . . . . .ha.
10p 2. Din cei 360 de elevi ai unei ]coli, 2 sunt @nscri]i la olimpiadele de limba
5
rom`n[ ]i matematic[, 5 din num[rul olimpicilor la limba rom`n[ ]i la matematic[
12
sunt @nscri]i la concursuri sportive, iar 3 din num[rul celor r[ma]i particip[ la olimpi-
4

Editura Cardinal 37
ada de limba englez[. C`\i elevi particip[ la fiecare activitate ]i c`\i nu particip[ la
nicio activitate?
10p 3. Verific[ dac[ forma ireductibil[ a frac\iei 3 n  3 n1  3n2 este 1 .
2n 2n1 2n2

9 9 9 7

Clasa a VI-a
Lucrarea 1 - (timp de lucru: 50 min.)
Alc[tuit[ de prof. CONSTANTIN BASARAB, Craiova

SUBIECTUL I.
1. Determina\i mul\imile: A  x  Z  x  2  x  2 ]i x  7;
B  x  Z  7  2x  1  10; C  x  Z  15  Z .
2x  3
2. Calcula\i: a) (15  9 )  (7  12 )  (10  8 ) : (3  6 );
b) (2 )  (3 )  (1 )  (8 )  2 9 : (4 ) .
3 2 8 7 15

3. Pre\ul unui obiect era 80 de lei ]i a crescut cu 10%, iar dup[ un timp a sc[zut
cu 25%.
a) Afla\i pre\ul final;
b) Cu c`te procente este mai mic pre\ul final dec`t cel ini\ial?
4. Triunghiul ABC este dreptunghic @n A ]i are m C  60  . Se duce CD bisec-
toarea unghiului C, D  AB ]i DE  BC, E  BC. Se cere:
a) m B ; b) CD  DB; c) DA  DE; d) AE  CD.
5. Se consider[ triunghiul isoscel ABC (AB  AC ). Perpendicularele @n B ]i C pe
AB, respectiv AC se intersecteaz[ @n P. Pe semidreapta BP se ia punctul M, astfel
@nc`t PM  BP. Perpendiculara @n M pe BM intersecteaz[ dreapta AC @n N.
Demonstra\i c[ CN  MN.

Lucrarea 2 - (timp de lucru: 50 min.)


Alc[tuit[ de prof. MARLENA BASARAB, Craiova

SUBIECTUL I.

1. Determina\i numerele @ntregi x ]i y, ]tiind c[ x  5  9 .


2 y3
2. Determina\i valorile @ntregi ale lui a pentru care
n 3 n 2 n n 2 n
an  2(1 )  (1 )  (1 )
n

a este num[r @ntreg oricare ar fi n  N.


3. Dac[ 0  x  y  z, calcula\i x  y  y  z  z  x  x  x .

38 Revista de matematic[ din Craiova


4. Fie triunghiul ABC cu m A  65  ]i m B  75  . Dac[ CD este bisectoarea
unghiului ACB, D  AB, DE  BC, E  AC ]i EF  AB, F  BC. Se cer m[surile un-
ghiurilor: CDE, ADE, EFC ]i DEF.
5. Se d[ triunghiul ABC cu m A  80  , Q  (BC ), QP  AB, P  (AB ) ]i QN 
 BC, N  AC. Dac[ m PQN  50  , demonstra\i c[ triunghiul ABC este isoscel.

Clasa a VII-a
Lucrarea 1 - (timp de lucru: 2 ore)
Alc[tuit[ de prof. DORINA DR{CEA, Craiova

SUBIECTUL I. (30 puncte) - Pe foaia de tez[ se trec doar rezultatele.


5p 1. Rezultatul calculului (2 3  75 ) : 7 . . . . . .
5p 2. Media geometric[ a num[rului a  12  3 ]i b  12  3 este egal[ cu
......
5p 3. Dac[ a  1 1
a  5, atunci: a  a 2  . . . . . .
2

5p 4. #n ABC dreptunghic, m A  90  , AD  BC, BD  6 cm, AD  12 cm.


Atunci: BC  . . . . . .cm.
5p 5. Aria discului cu raza de 8 cm este egal[ cu . . . . . .cm 2 .
5p 6. Triunghiul ABC este echilateral cu aria egal[ cu 9 3 cm 2 . Atunci:
P ABC  . . . . . .cm.

SUBIECTUL II. (30 puncte) - Pe foaia de tez[ se trec rezolv[rile complete.

10p 1. Descompune\i @n factori:


a) (2x  3 )  8x  12; b) x 2  x  12; .
2

c) 2  x  (x  2 ) ; d) 49  x 2  7ax  ax 2 .
3

10p 2. Rezolva\i ecua\ia: (x  2)  (x  3 )  (x  1)  2(x  2)  5.


2 2

10p 3. Afla\i dou[ numere, ]tiind c[ unul dintre ele este cu 44 mai mare dec`t
cel[lalt ]i c[ @mp[r\ind suma lor la diferen\a lor, ob\inem 46 rest 26.

SUBIECTUL III. (30 puncte) - Pe foaia de tez[ se trec rezolv[rile complete.


15p 1. Fie ABCD un trapez isoscel cu AB  CD, AC  BD, AB  20 cm, CD  12 cm
]i AC  BD  O. Calcula\i:
a) #n[l\imea trapezului; b) Aria trapezului;
c) Perimetrul trapezului; d) A DOC .

Editura Cardinal 39
15p 2. #n triunghiul dreptunghic ABC, m A  90  , B  2 ]i AB  10 5 .
3
a) Afla\i aria ]i perimetrul triunghiului.
b) Calcula\i lungimea @n[l\imii AD ]i lungimile proiec\iilor catetelor pe ipo-
tenuz[.

Lucrarea 2 - (timp de lucru: 50 min.)


Alc[tuit[ de prof. DORINA DR{CEA, Craiova
1
1. Calcula\i: a) 2  3 5 3  2 .
75 12 3

b) (2 3  1 )  ( 3  2 )  (5 3  1 )  (5 3  1 ).
2 2

c) (3x  1)  (x  2 )  3(x  1 )  2(x  3 ) .


2 2

2. a) Descompune\i @n factori: a 2  b 2  3a  3b ]i x 2  y 3  10y  25.


b) Dac[ a  1a  10, calcula\i a  a 4 .
4 1
c) Demonstra\i c[ pentru orice numere reale x ]i y, x 2  y 2  4x  10y  30  0
3. Dup[ ce a parcurs 15 km, un biciclist constat[ c[ mai mare de parcurs 75%
din distan\[. Ce distan\[ are de parcurs biciclistul?
4. Fie ABCD un trapez dreptunghic, AB  AC, m A  m D  90  , AC  BC ]i
AD  BC  M. Dac[ DC  9 cm ]i @n[l\imea trapezului are 12 cm, calcula\i:
a) Aria ]i perimetrul trapezului; b) Aria ]i perimetrul  MAB.
5. #n cercul de centru O ]i raza 10 cm, fie A ]i B dou[ puncte diametral opuse ]i
AC o coard[ situat[ la o distan\[ de 5 cm de centrul cercului. Calcula\i aria, perime-
trul ]i m[surile unghiurilor  ABC, precum ]i lungimile arcelor AC ]i BC.

Clasa a VIII-a
Lucrarea 1 - (timp de lucru: 120 min.)
Alc[tuit[ de prof. LAVINIA DALIE, Certeju de Sus, Hunedoara

SUBIECTUL I. (30 puncte) - Pe foaia de tez[ se trec numai rezultatele.


1
5p 1. Rezultatul calculului  1  1 1 este egal cu . . . . . .
2
5p 2. #ntr-o urn[ sunt 4 bile albe ]i 6 bile negre. Se extrage o bil[. Care este
probabilitatea ca bila extras[ sa fie alb[?
5p 3. Diferen\a dintre cel mai mic num[r @ntreg ]i cel mai mare num[r @ntreg, din
intervalul 4; 6 este egal cu . . . . . .

40 Revista de matematic[ din Craiova


5p 4. Un cerc are aria ]i lungimea exprimate prin acela]i num[r. Diametrul cercului
este egal cu . . . . . . cm.
5p 5. Dac[ ABCD este un tetraedru regulat, atunci m[sura unghiului dintre
muchiile AB ]i CD este egal[ cu . . . . . . grade.
5p 6. #n tabelul de mai jos este prezentat[ situa\ia mediilor ob\inute de elevii unei
]coli la matematic[:
Media 1  3 99 4  4 99 5  5 99 6  6 99 7  7 99 8  8 99 9  10
Num[r
6 9 10 6 17 23 4
elevi
Num[rul elevilor care au promovat este egal cu . . . . . .

SUBIECTUL II. (30 puncte) - Pe foaia de tez[ se trec rezolv[rile complete.

5p 1. Desena\i un trunchi de con care are sec\iunea axial[ trapezul ABCD cu baza
mare CD.
5p 2. Dac[ a  1  1 ]i b  1  1 , ar[ta\i c[ a  b.
3  2 2 3 3  2 2 3
5p 3. Un automobil parcurge distan\a dintre dou[ localit[\i @n trei zile astfel: @n
prima zi parcurge 5 din distan\[, a doua zi parcurge 1 , din distan\a r[mas[, iar a treia
6 3
zi este tractat pe distan\a de 100 km ca s[ poat[ ajunge la destina\ie. Care este distan\a
dintre cele dou[ localit[\i?
4. Se consider[ func\ia f : R R, f (x)  2x  6.
5p a) Calcula\i f (2 )  f (2 ).
5p b) Dac[ A ]i B sunt punctele de intersec\ie a reprezent[rii grafice a func\iei
f cu axele de coordonate Ox ]i respectiv Oy, determina\i coordonatele unui punct C
situat pe axa Ox, astfel @nc`t triunghiul ABC s[ fie dreptunghic @n punctul B.
5. Ar[ta\i c[ E(x )  2 x  9  x  2  x 2 5x  2  1, pentru orice x  R 
2 2
5p
x  4x  3 x  1 x 1
1, 1, 3.

SUBIECTUL III. (30 puncte) - Pe foaia de tez[ se trec rezolv[rile complete.

1. Terenul de sport al unei ]coli are form[ de p[trat cu perimetrul egal cu 240 m.
Terenul este @nconjurat de o pist[ cu l[\imea de 3 m. Calcula\i:
5p a) Aria terenului de sport.
5p b) Aria pistei.
5p c) Perimetrul exterior al pistei.
2. Un bazin @n form[ de paralelipiped dreptunghic ABCD A  B  C  D  cu baza
ABCD are AB  10 m, BC  7 m ]i AA   3 m.
5p a) Calcula\i volumul bazinului.
5p b) #n bazin sunt 105000 litri de ap[. Determina\i @n[l\imea apei din bazin.

Editura Cardinal 41
5p c) #n bazin sunt 105000 litri de ap[. Golirea bazinului se face prin 3 robinete,
fiecare cu un debit de 10 litri pe secund[. #n c`t timp se gole]te bazinul?

Lucrarea 2- (timp de lucru: 120 min)


Alc[tuit[ de prof. SIDONIA CAZACU, Craiova

SUBIECTUL I. (30 puncte) - Pe foaia de tez[ se trec numai rezultatele.


5p 1. Rezultatul calculului 4  5  (3 ) este egal cu . . . . . .
5p 2. 6 muncitori termin[ o lucrare @n 6 zile. C`\i muncitori ar termina lucrarea
respectiv[ @n 18 zile?
5p 3. Suma numerelor naturale din intervalul 6; 4 este egal[ cu . . . . . .
5p 4. Aria unui romb cu latura de 4 cm ]i un unghi de 150  este egal[ cu . . . . . . cm
2
.
5p 5. Un con circular drept are generatoarea de 8 cm ]i raza bazei egal[ cu 4 cm.
M[sura unghiului format de generatoare cu planul bazei este egal[ cu . . . . . . grade.
5p 6. #n diagrama de mai jos sunt prezentate preferin\ele celor 320 de elevi ai unei
]coli pentru limbile str[ine care se studiaz[ @n ]coal[.

25 Num[rul elevilor care prefer[ limba francez[ este


gerlimb% egal cu . . . . . .
ma a 60%
n[
limba
limba englez[
francez[

SUBIECTUL II. (30 puncte) - Pe foaia de tez[ se trec rezolv[rile complete.

5p 1. Desena\i pe foaia de examen un cub ABCD A  B  C  D  .


5p 2. Dac[ a  3, unde a, b sunt numere reale nenule, calcula\i 3a  2b .
b 5b
5p 3. Dup[ ce a parcurs 1 din lungimea unui drum, un turist constat[ c[ dac[ mai
4
parcurge 85 km, el dep[]e]te cu 10 km jum[tate din lungimea drumului. Afla\i lungi-
mea drumului.
4. Se consider[ func\ia f : R R, f (x)  2x  4.
5p a) Reprezenta\i grafic func\ia.
5p b) Determina\i distan\a de la punctul A(5, 8 ) la reprezentarea grafic[ a
func\iei.
: x 2 x  2 , unde
2
5p 5. Se consider[ expresia E(x )  x  2
x1 1x x 1
x  R 1, 0, 1. Ar[ta\i c[ E(x )  1.

42 Revista de matematic[ din Craiova


SUBIECTUL III. (30 puncte) - Pe foaia de tez[ se trec rezolv[rile complete.

1. #n figura al[turat[ este reprezentat[ o piscin[ (p[tratul ABCD) ]i o zon[ de


relaxare cu ]ezlonguri (triunghiul echilateral BCE) cu BC  30 m.

5p a) Calcula\i aria piscinei. A B


5p b) Ar[ta\i c[ distan\a de la orice
@not[tor din piscin[ la un ]ezlong din zona
de relaxare este mai mic[ de 60 m. E
5p c) C`te ]ezlonguri se pot a]eza
(num[r maxim) @n zona de relaxare, dac[
pentru un ]ezlong sunt necesari 2 m 2 spa\iu
(se consider[ 3  1, 73.). D C

2. Prisma hexagonal[ regulat[ dreapt[ ABCDEF A  B  C  D  E  F  are aria bazei


egal[ cu 54 3 cm2 ]i toate muchiile sunt congruente.
5p a) Ar[ta\i c[ AA   6 cm.
5p b) Determina\i m[sura unghiului dintre dreptele AC  ]i DF  .
5p c) Calcula\i distan\a de la punctul A la planul (CDB  ).

Clasa a IX-a
Lucrarea 1 - (timp de lucru: 90 min.)
Alc[tuit[ de prof. EUGENIA TEREUJANU, Craiova

SUBIECTUL I.
1. Dac[ a  2, rezolva\i @n R ecua\ia x 3  2ax 2  (a 2  1 )x  2  2a  0 ]i scrie\i
r[d[cinile @n ordine cresc[toare.
2. Determina\i m  R, astfel @nc`t ecua\ia x(x  1 )(x  2 )(x  3 )  m s[ aib[ toate
r[d[cinile reale.
3. Studia\i monotonia func\iei: f : (0,  )  R, f (x )  x  1x .

SUBIECTUL II.
1. Laturile unui triunghi au lungimile mai mici ca 1. Ar[ta\i c[ aria triunghiului
3
este mai mic[ dec`t .
4
2. Dreapta AD, D  BC, @mparte triunghiul ABC @n dou[ triunghiuri. Fie r 1
raza cercului @nscris @n ABD ]i r 2 raza cercului @nscris @n ADC. Dac[ r este raza

Editura Cardinal 43
cercului @nscris ABC s[ se arate c[ r  r 1  r 2 .
3. Determina\i numerele reale x pentru care: 1  x 2  4x 3  3x.
4. Rezolva\i sistemele:
 2x  x 2 y  y
  4xy(2x 2  1 )  1
a)  2y  y 2 z  z ; b)  .
 2z  z 2 x  x  x2  y2  1


Lucrarea 2 - (timp de lucru: 90 min.)


Alc[tuit[ de prof. OANA DOVAN, Craiova

SUBIECTUL I.

1. Afla\i perimetrul real a  R cu proprietatea:


f a (x )  2x 2  (a  2 )x  a  1  0, ( ) x  1, 1.
2. Rezolva\i @n R: 4x 2  3x  15  4x 2  3x  8  7.

SUBIECTUL II.

1. Fie ABC un triunghi de laturi a, b, c ]i I centrul cercului circumscris acestuia.


Ar[ta\i c[ oricare ar fi M din planul triunghiului ABC, avem:

 a 
 b  c 
MJ  MA  MB  MC.
abc abc abc
2. Determina\i x1 , x2 , , xn  R, (n  N , fixat ), ]tiind c[:
x  x2    xn
x1  1  2 x2  22  3 x3  32    n xn  n2  1 .
2
3. Reprezenta\i grafic func\ia: f : R  R, f (x )  sin . x
2

Clasa a X-a
Lucrarea 1 - (timp de lucru: 90 min.)
Alc[tuit[ de prof. C{T{LIN CRISTEA, Craiova

1. a) S[ se determine c`te numere de 3 cifre distincte se pot forma cu elemen-


tele mul\imii 0, 1, 2, 3, 4, 5.
b) S[ se calculeze probabilitatea ca o func\ie f definit[ pe mul\imea 2, 4, 6,
8 cu valori @n mul\imea 1, 3, 5, 7, 9 s[ fie injectiv[.
c) #ntr-o urn[ se afl[ 3 bile albe, 5 bile negre ]i 7 bile ro]ii. Se extrag simul-
tan din urn[ 2 bile.

44 Revista de matematic[ din Craiova


i) Care este probabilitatea ca ambele bile extrase s[ fie albe?
ii) Care este probabilitatea ca o bil[ s[ fie alb[ ]i una neagr[?
2. a) S[ se rezolve ecua\ia: 19  A 4x4  30  C x9x3 .
b) Care este valoarea maxim[ a func\iei f (x )  C x8x12 pe domeniul ei de
2

defini\ie?
n

c) Se d[ binomul 1 x 2lg x
, n  N  , x  0, av`nd suma coefici-
x2  3 x2
en\ilor binomiali egal[ cu 256. S[ se determine x ]tiind c[ al ]aselea termen al dez-
volt[rii este 5600.
3. Se consider[ punctele A(0, 1 ), B(1, 1 ), C(1, 3 ).
a) Calcula\i aria triunghiului ABC.


b) Calcula\i modulul vectorului AB.
c) Calcula\i cos (ABC ).
4. #n sistemul cartezian de coordonate xOy se consider[ punctele A(3, 0 ) ]i
B(0, 2 ).
a) Scrie\i ecua\ia mediatoarei d a segmentului AB.
b) S[ se determine coordonatele urm[toarelor puncte: E  d  OB, F 
 d  OA, M mijlocul segmentului AE, N mijlocul segmentului BF.
c) S[ se verifice dac[ dreptele MD ]i ND sunt perpendiculare, unde D este
mijlocul segmentului AB.

Lucrarea 2 - (timp de lucru: 90 min.)


Alc[tuit[ de prof. MIHAELA ST{NCELE, Craiova

1. Rezolva\i ecua\iile:
a) 3(x  2 )!  (x  3 )!; b) C 2x3  21;
(
c) C44x9
x1)
 5A34x7 .
n

2. Se consider[ dezvoltarea y  1 , y  R  ]i n  N .
24y
a) S[ se determine n pentru ca coeficien\ii termenilor 1, 2 respectiv 3 ai dez-
volt[rii formeaz[ o progresie aritmetic[.
b) Pentru n  8 s[ se g[seasc[ termenii dezvolt[rii care @l con\in pe y.
c) Calcula\i suma coeficien\ilor dezvolt[rii.
3. Se dau A(1, 1 ) ]i B(2, 3 ) ]i dreapta d : x  4y  7  0.
a) S[ se determine ecua\ia dreptei AB.
b) S[ se determine coordonatele punctului C  d, astfel @nc`t triunghiul ABC
s[ fie dreptunghic @n C.
c) S[ se determine ecua\ia mediatoarei dreptei AB.
4. Demonstra\i egalit[\ile:
a) C 07  2C 17  2 2 C 27    2 7 C 77  3 7 .
b) C04  3 C14  3C24  3 3 C34  9C44  (1  3 ) .
4

Editura Cardinal 45
Clasa a XI-a
Lucrarea 1 - (timp de lucru: 90 min.)
Alc[tuit[ de prof. C{T{LIN CRISTEA, Craiova

SUBIECTUL I.

 xyz2

1. Se consider[ sistemul:  mx  y  z  0, m  R .
 x  2y  mz  1

a) Ar[ta\i c[ determinantul matricei sistemului este egal cu m 2  3m  4.
b) Determina\i m  R pentru care sistemul este compatibil determinat.
c) Rezolva\i sistemul pentru m  1.

SUBIECTUL II.

1. Calcula\i:
 cos x  ln(1  2017x )  3 x
a) lim .
x0 sin 2x
lim 2x  4x 2  3x  2 .
b) x

x3  x  1  x2
c) lim .
x
x  3 x4  x  1
2. Fie func\ia f : D  R, f (x )  x  ax 2  bx  1 , a, b  R, a  0, unde D este
domeniul de defini\ie al func\iei f.
a) Pentru a  b  1, calcula\i f  (x), x  R.
b) Pentru a  b  1, determina\i asimptotele la graficul func\iei f.
c) Determina\i a, b  R,astfel @nc`t lim f (x )   1 .
x 2

46 Revista de matematic[ din Craiova


Lucrarea 2 - (timp de lucru: 90 min.)
Alc[tuit[ de prof. C{T{LIN CRISTEA, Craiova

SUBIECTUL I.
 2x  y  z  4

 x  y  z  m
1. Se consider[ sistemul:  , m  R.
 3x  y  2z  6

 x  my  z  m
a) Ar[ta\i c[ sistemul nu poate avea solu\ia x 0  1, y 0  1, z 0  1 pentru
nicio valoare a lui m.
b) Determina\i valorile lui m  R pentru care sistemul este compatibil.
c) Rezolva\i sistemul pentru m  0.

SUBIECTUL II.

1. Calcula\i:
a) lim 2  3    10  9 ;
x x x 1
b) lim (1  sin 2 x ) tg 2 x .
x0 sin 11x x0

2. Se d[ func\ia f : (0,  )  R, f : (1, 0 )  (1,  ), f (x )  ln 22x .


x 1
a) Calcula\i f  (1 ).
b) Scrie\i ecua\ia tangentei la graficul func\iei f @n punctul de abscis[ x 0  1.
c) Determina\i asimptotele verticale la graficul func\iei f.
d) Studia\i monotonia func\iei f.

Editura Cardinal 47
Clasa a XII-a
Lucrarea 1 - (timp de lucru: 2 ore) - Se acord[ 10 puncte din oficiu.
Alc[tuit[ de prof. DANIEL CRISTIAN CIURCEA, Craiova

SUBIECTUL I (30 puncte)

5p 1. Calcula\i modulul num[rului complex z  1  i  i 2    i 2017 .


5p 2. Demonstra\i c[ graficul func\iei f : R R, f (x)  x2  2x  3 este situat @n @n-
tregime deasupra axei Ox.
5p 3. Rezolva\i ecua\ia 3 x  3 x2  270.
5p 4. Afla\i num[rul elementelor mul\imii ab  a, b  N, ab7.

  
5p 5. Fie un triunghi ABC ]i punctele M, N, P definite prin rela\iile CA  2MA,

     
BN  NM, 2BP  PC. Ar[ta\i c[ punctele A, N, P sunt coliniare.
6  2
5p 6. Demonstra\i c[ sin 105   .
4
SUBIECTUL II (30 puncte)

1 2 3 4 5 6 1 2 3 4 5 6
1. Fie permutarea    S 6 ]i   .
3 5 4 6 2 1 2 4 6 1 5 3
5p a)Calcula\i  2017 .
5p b) Rezolva\i ecua\ia x  .
c) Stabili\i dac[ exist[ n  N , pentru care  n n1  .
2
5p

2. Pe R definim opera\ia x  y  2xy  2x  2y  3.


5p a) Ar[ta\i c[ G  (1,  ) este parte stabil[ a lui R @n raport cu opera\ia dat[.
5p b) Calcula\i (2017 )  (2016 )    0  1    2017.
5p c) Rezolva\i ecua\ia x  x  x  344.

SUBIECTUL III (30 puncte)

1. Fie f : (0, ) R, f (x)  ln x  arctg x.


5p a) Calcula\i f  (x ), x  (0,  ).
5p b) Determina\i asimptotele graficului func\iei f.
5p c) Demonstra\i c[ f este bijectiv[.

48 Revista de matematic[ din Craiova


1
2. Fie ]irul I n  x n dx, n  N.
0 2x 1
5p a) Calcula\i I 0 , I 1 .
5p b) Studia\i monotonia ]irului (I n ).
5p c) Calcula\i lim
n
(nI n ).

Lucrarea 2 - (timp de lucru: 2 ore) - Se acord[ 10 puncte din oficiu.

Alc[tuit[ de prof. DANIEL CRISTIAN CIURCEA, Craiova

SUBIECTUL I (30 puncte)

1. Calcula\i 5  ( 3  2 ) .
2
5p
5p 2. Afla\i coordonatele punctului de intersec\ie al graficelor func\iilor
f, g : R  R, f (x )  2x  5; g(x )  x  8.
5p 3. Rezolva\i ecua\ia 9 x  3 x  6  0.
5p 4. Afla\i c`te numere de trei cifre, toate pare, exist[.
 
 
5p 5. Fie un triunghi ABC ]i punctele M, N definite de rela\iile 2BM  MC ]i

 
 
      
2BC  3NB. Ar[ta\i c[ AM  AN  AB  AC.
5p 6. }tiind c[   R astfel @nc`t sin   cos   5 , calcula\i sin(2 ).
4

SUBIECTUL II (30 puncte)

1 n
1. Fie matricele A(n )  , n  N.
0 1
5p a) Rezolva\i ecua\ia A(2 )  x  A(4 ).
n
5p b) Calcula\i (A(1)) , n  N .
5p c) Calcula\i A(1 )  A 2 (1 )  A 3 (1 )    A n (1 ).

6 4
2. Fie A   M 2 (R ) ]i mul\imea G  x(t )  I 2  t  A  t  Z.
9 6
5p a) Demonstra\i c[ G este parte stabil[ a lui M 2 (Z ) @n raport cu opera\ia de
@nmul\ire a matricelor.
5p b) Demonstra\i c[ (G,  ) este grup abelian.
5p c) Calcula\i x(1  2)  x(2  3)  x(3  4)    x(n(n  1)), n  N .

Editura Cardinal 49
SUBIECTUL II (30 puncte)

1. Fie f : R R, f (x)  3x  ax  1, a  R.


f (x )
5p a) Afla\i valoarea lui a pentru care lim
x0 x  ln 6.
5p b) Afla\i valoarea lui a pentru care f (x)  0, () x  R.
5p c) Pentru a  0 scrie\i ecua\ia tangentei la grafic dus[ @n punctul de abscis[ 0.

2. Fie f, g : 0,   R, f (x )  sin x ; g(x )  cos x .


2 (sin x  cos x ) 2 (sin x  cos x ) 2

2
5p a) Ar[ta\i c[  (f (x )  g(x ))dx  0.
0

2
5p b) Calcula\i 
sin x
1
 cos x
dx.
0
5p c) Determina\i aria subgraficului func\iei f.

*
* *

50 Revista de matematic[ din Craiova


************************************************************ *********
* PROBLEME PROPUSE *
*********************************************************************

GIMNAZIU

Clasa a V-a
G.1233. Determina\i numerele prime a  b  c care verific[ egalitatea
a 3  b 3  c 3  7210.
Prof. CERASELA BOCIU, Timi]oara

G.1234. Determina\i numerele naturale x, astfel @nc`t ultima cifr[ a num[rului n 


 x!  87 s[ fie 7, unde x!  1  2    x, x  N ]i 0!  1.
Prof. DOINA STOICA ]i MIRCEA MARIO STOICA, Arad
G.1235. Se d[ rela\ia 7a  12b  2018 unde a, b  N.
a) Este posibil ca a  b  113?
b) Determina\i a ]i b dac[ a  b  214.
Prof. CONSTANTINA PRUNARU, Craiova
Prof. LUIZA CREMENEANU, Craiova
G.1236. Ar[ta\i c[ num[rul x  1  2  3  3  4  5    1961  1962    3821 nu
este p[trat perfect.
Prof. MIRCEA MARIO STOICA, Arad

G.1237. Exist[ numere naturale n astfel @nc`t 8n  1 ]i 6n  4 s[ fie ambele p[trate


perfecte?
Prof. ALECU ORLANDO, Ro]iorii de Vede, Teleorman

_______________________________________
Se primesc solu\ii p`n[ la 15.07.2017 (data po]tei)

Editura Cardinal 51
Clasa a VI-a
G.1238. Fie x un num[r prim diferit de 7. Determina\i valorile lui x pentru care
frac\ia 1 are o perioad[ format[ din 6 cifre.
7x
Prof. VALERICA POMETESCU, Craiova
G.1239. Determina\i numerele naturale x, y ]i z, ]tiind c[ @ndeplinesc simultan
y  961
condi\iile: 1) x  961   7 . 2) x  y  2600.
50 80 25
Prof. MIRCEA MARIO STOICA, Arad
G.1240. Ar[ta\i c[ restul @mp[r\irii num[rului a  n 2017  2017n  2017 la 3 este 1,
oricare ar fi num[rul natural n.
Prof. CERASELA BOCIU, Timi]oara
G.1241. Fie triunghiul ABC cu AB  AC, m(ABC )  40  . O perpendicular[ dus[
pe bisectoarea AD a unghiului ABC, D  (BC ), intersecteaz[ pe BC @n M, astfel @nc`t
MB  AB  AC, B  (MC ). S[ se calculeze m[surile unghiurilor triunghiului ABC.
Prof. VALERICA POMETESCU, Craiova
2 2 2
G.1242. Dac[ a, b, c  N  ]i a  bc  ca , demonstra\i c[ a  b  c.
b
Prof. TATIANA CRISTEA, Craiova
Prof. LUMINI|A MIHALACHE, Craiova

Clasa a VII-a
G.1243. Numerele @ntregi a, b, c verific[ rela\ia 3c 2  1  2(3bc  2ab  ac ).
Calcula\i a  b  c.
Prof. CERASELA BOCIU, Timi]oara

G.1244. Ar[ta\i c[ 1  1  1  51 .


1  51 2  50 51  1 26
Prof. MIRCEA MARIO STOICA, Arad
G.1245. Fie a, b, c, x, y, z  R, astfel @nc`t (a  b  c ) 2  3(ab  bc  ca  x 2  y 2  z 2 ).
Ar[ta\i c[ a  b  c ]i x  y  z  0.
Prof. CARMEN TERHECI, Craiova
Prof. CAMELIA DAN{, Craiova
G.1246.Afla\i perechile de numere naturale k ]i n pentru care (n  1 )(n  3 )  2 k  7.
Elev[ HARICLEA BODROGEAN, Bucure]ti

52 Revista de matematic[ din Craiova


G.1247. Rezolva\i @n Z ecua\ia: x(x  9 )  y 2 (y 2  5 ).
Prof. MIHAELA BERINDEANU, Bucure]ti

Clasa a VIII-a
G.1248. Fie punctele A(7; 3 ) ]i B(4; 6 ) @ntr-un sistem de axe perpendiculare.
Determina\i pozi\ia punctului C situat pe axa absciselor, astfel @nc`t triunghiul ABC s[
fie dreptunghic @n C.
Prof. DOINA STOICA ]i MIRCEA MARIO STOICA, Arad
G.1249. Fie a, b  0, , astfel @nc`t a  b  2. Ar[ta\i c[ (ab )
2015
(a 2  b 2 )  2.
#n ce caz avem egalitate?
Prof. ROXANA VASILE ]i CRISTINA BONDOC, Craiova
G.1250. Fie cubul ABCDA  B  C  D  de muchie a, punctele M ]i N mijloacele
muchiilor AB, respectiv BC, iar AN  DM  P. Calcula\i:
a) Tangenta unghiului dintre planele (PA  D  ) ]i (ABC ).
b) Distan\a de la punctul C  la dreapta AN.
Prof. CERASELA BOCIU, Timi]oara
G.1251. Ar[ta\i c[ num[rul N  3 4 2017
4 3 2017
admite doi divizori mai mari ca 2017.
Prof. MIHAELA BERINDEANU, Bucure]ti
G.1252. Fie a, b, c, d  R, astfel @nc`t abcd  1. Demonstra\i c[

N 2016  a  2016  b  2016  c  2016  d


1  4  ab  abc 1  b  bc  bcd 1  c  cd  cda 1  d  ad  abd
este un num[r natural prim.

Elev[ ELIZA CONSTANTINESCU, Bucure]ti

Editura Cardinal 53
LICEU

Clasa a IX-a
L.1253. S[ se rezolve ecua\ia x  x  7  x.
Prof. MIRCEA MARIO STOICA, Arad
 3
L.1254. Ar[ta\i c[ sin  sin 1  1 .
2 2 8
Prof. LAUREN|IU MOLDOVAN, Bucure]ti
Prof. MARIAN VOINEA, Bucure]ti
2 2 xy(x 2  y 2 ) x 2  y 2
L.1255. Fie x, y  (0,  ), x  y. Ar[ta\i c[  .
2xy  x 2  y 2 2
#n ce caz avem egalitate? Interpretare geometric[.
Prof. LUCIAN TU|ESCU, Craiova
Prof. VICTOR NICOLAE, Bucure]ti
L.1256. Dac[ a, b, c, d, e  R cu proprietatea a  b  c  d  e  abcde, demonstra\i
c[: a 4  b 4  c 4  d 4  e 4  abcde.
Prof. LUCIAN TU|ESCU, Craiova
Prof. CARMEN TERHECI, Craiova
L.1257. Se consider[ numerele reale a 1 , a 2 , , a 2017 @n progresie aritmetic[ cu ra\ia
pozitiv[ ]i a 1  1. Demonstra\i c[:
4030 a3 a4 a 2017 1 1 1
a 1  a 2015  a 1 a 2  a 2 a 3    a 2015 a 2016  1  a 1  a 2    a 2015 .
Prof. DAN SECL{MAN, Craiova

Clasa a X-a
L.1258. Rezolva\i ecua\ia: log 6 x  4 x  log 2 4 x .
Prof. SIMONA RADU, Craiova
Prof. SIMONA MIU, Craiova
L.1259. It ak  N  1, k  1, 2, , n, then log na1 (a 2 ! )  log na 2 (a 3 ! )   
n
 log na n (a 1 ! )  n  (a k  1 ).
k1
Prof. MIHA’LY BENCZE, Bucure]ti

54 Revista de matematic[ din Craiova


L.1260. Dac[ n  N, n  3, A  1, 2, 3, , n s[ se determine num[rul submul\i-
milor cu 3 elemente ale lui A, cu proprietatea c[ produsul acelor elemente ale submul-
\imii s[ fie divizibil cu 4.
Prof. DAN NEDEIANU, Dr. Tr. Severin
L.1261. Demonstra\i c[ @ntr-un triunghi ABC avem m(AIO )  90  , dac[ ]i numai
dac[ AB  AC  3BC.
Prof. FLORIN ST{NESCU, G[e]ti, D`mbovi\a
L.1262. Dac[ a, b, c, d  (1,  ), demonstra\i c[:
log ab (c  d )  log bc (d  a )  log cd (a  b )  log da (b  c )  2 1  4  log abcd 2 .
Preciza\i c`nd are loc egalitatea.
Prof. DAN SECL{MAN, Craiova

Clasa a XI-a
L.1263. It x n1  x n  x 2017
n , x 1  (0, 1 ) for all n  N .
1) Prove that (x n ) n1 is convergent and compute it’s limit.
2) Compute lim n
(x 2017
1  x 2017
2    x 2017
n ).
3) Compute lim 2016 n  x .
n n

Prof. MIHA’LY BENCZE, Bucure]ti


ln x
n
L.1264. S[ se calculeze lim x  ln x .
n ln(x  ln x )
Prof. IONEL TUDOR, C[lug[reni, Giurgiu
L.1265. Fie A, B  M 3 (R ) ]i F mul\imea tuturor func\iilor f : R R ce au urm[-
toarele 5 propriet[\i: 1) f (x)  f (y)  f (x  y), () x, y  N;
2) f (0 )  f (1 )  det A;
3) f (x)  f (x)  dAB12 , () x  R;
4) f x  det a  xB), () x  R Z;
( ) (
5) Ecua\ia f (x )  0 are exact 3 solu\ii @n mul\imea R Z.
Determina\i card F.
Nota\ie: d AB
12 reprezint[ suma determinan\ilor ce au o coloan[ din A ]i dou[ coloane
din B.
Elev LUIGI-IONU| CATANA , Potcoava, Olt
L.1266. Fie n  N, n  2, fixat ]i A, B  Mn (R) cu proprietatea c[ exist[ a, b  R ,
a  b, astfel @nc`t aA  bB  (a  b )AB.
a) Ar[ta\i c[ AB  BA iar matricele U  (a  b )A  bI n ]i V  (a  b )B  aI n
sunt inversabile ]i @n plus U 1  1  V.
ab

Editura Cardinal 55
b) Dac[ matricele I n  A ]i I n  B sunt inversabile, ar[ta\i c[ A este inversabil[
dac[ ]i numai dac[ B este inversabil[ ]i @n plus bA 1  aB 1  (a  b )I n .
Prof. DAN SECL{MAN, Craiova
1
L.1267. Calcula\i: lim
n
n n 2017 n  1  ln 2017 .
Prof. ILEANA DIDU, Craiova
Prof. CAMELIA DAN{, Craiova

Clasa a XII-a
L.1268. Fie   1. S[ se calculeze:  x4  2
x 1 dx, x  R.
31

x  3x  1
Prof. NICOLETA BRAN, Craiova
Prof. AUREL OPORANU, Craiova
L.1269. Dac[ f : R R este o func\ie continu[, ar[ta\i c[:
n n n
2
f ( nx ) 4
f ( nx ) 2n
f ( nx )
lim
n
n x2
dx  
n x2
dx    n x2
dx  f (0 ).
3 5 2n1

Prof. FLORIN ST{NESCU, G[e]ti, D`mbovi\a


L.1270. Determina\i P  R x, astfel @nc`t P(x2 )  x(x  1)P(x), () x  R.
Prof. PETRI}OR ROC}OREANU, Craiova
L.1271. Fie f : 0, 1  R o func\ie care admite primitive cu proprietatea c[ pentru
orice primitiv[ F a sa exist[ rela\ia F(1 )  F(0 )  1 . Demonstra\i c[:
2
a) Exist[ a  (0, 1 ) astfel @nc`t f (a )  a.
b) Pentru orice n  N , exist[ c 1 , c 2 , , c n  (0, 1 ), c 1  c 2    c n ,
cu proprietatea f (c 1 )  f (c 1 )    f (c n )  n .
2
Prof. DAN SECL{MAN, Craiova

e  4e  e 2  2 1 x 2 e  4e  e 2  2
L.1272. Demonstra\i c[:  e dx  .
2 0 2
Prof. ION NEDELCU, Ploie]ti
Prof. AUREL CHIRI|{, Slatina

*
* *

56 Revista de matematic[ din Craiova


RUBRICA REZOLVITORILOR
B{BENI (jud. V`lcea): }coala Gim. “Rom`ni”, clasa a V-a, prof. DOINA-MARIA
POPESCU: V[caru }tefania (G 1193-1197).
CRAIOVA (jud. Dolj): Lic. Teoretic “Tudor Arghezi”, clasa a V-a, prof.
DORINA DR{CEA: Ro]ca Izabela (G 1193-1197), Nicola Lucian (G 1193-1197),
Stancu }erban-Andrei (G 1193-1197), Nicolae Ana-Maria (G 1193-1197), clasa a
VI-a, prof. DORINA DR{CEA: Tra]c[ Mihai (G 1198-1202), M[rinic[ R[zvan (G
1198-1202), Andri\oiu Mihai (G 1198-1202), }erban Alex (G 1198-1202), Preda
Mihai (G 1198-1202), P@rvu Darius (G 1198-1202), Mitrana Andreea (G
1198-1202), clasa a VII-a, prof. DORINA DR{CEA: Cojoc[reanu Ania (G
1203-1207), Fota Alexandra (G 1203-1207), T[n[sescu Daniela (G 1203-1207),
Popescu Sorin (G 1203-1207), Mitrache Teodora (G 1203-1207), Dragomir Anda
(G 1203-1207), clasa a VIII-a, prof. DORINA DR{CEA: Belciug Ianis (G
1208-1212), Modi Mihai (G 1208-1212), Ivanciu Alex (G 1208-1212), Sandu
Alexia (G 1208-1212), P@rvulescu Roxana (G 1208-1212), Buia Viviana (G
1208-1212), Iag[ru Adina (G 1208-1212), Gruia Robert (G 1208-1212),
Gr[unteanu Cristina (G 1208-1212), Br[tan Alex (G 1208-1212), Li\u R[zvan (G
1208-1212), Cre\u Maria (G 1208-1212), C[l[ra]u Maria (G 1208-1212), Mitran
Rebeca (G 1208-1212), Ionescu Andrei (G 1208-1212), Gruescu Riana (G
1208-1212). C.N. Militar “Tudor Vladimirescu”, clasa a IX-a, prof. MIHAELA
MIREA: Diaconu Ciprian (L 1213-1217), M`r]anu Alexandru (L 1213-1217),
clasa a IX-a, prof. ANICU|A BE|IU: Sp[taru Patricia (L 1213-1217), clasa a X-a,
prof. MIHAELA MIREA: Amzuloiu Teodor (L 1218-1222), Vlad Cezara (L
1218-1222), clasa a XI-a, prof. ANICU|A BE|IU: Cojocaru Marian (L 1223-1227),
Barbu Cornel (L 1223-1227). C.N. “Fra\ii Buze]ti”, clasa I, prof. @nv. primar
C{T{LINA DURLEA ]i ANGELA POPESCU: Ignat David (I 1-25), Tra]c[ Gloria
(I 1-25), Mogoant[ Corina (I 1-25), clasa a V-a, prof. MARIA IONESCU:
Cr[ciunescu Matei (G 1193-1197), clasa a VI-a, prof. CLAUDIU CIULCU:
Geampalia Alexia (G 1198-1202), Jilavu Izabela (G 1198-1202), V[ileanu Brianna
(G 1198-1202), clasa a VII-a, prof. MARIA IONESCU: Ciuperceanu Vlad (G
1203-1207), clasa a IX-a, prof. TEODORA R{DULESCU: Sandu George (L
1213-1217), clasa a IX-a, prof. LUCIAN TU|ESCU: Dinc[ Maria (L 1213-1217),
Toporan Laura (L 1213-1217), Ilea Mihai (L 1213-1217), Cr[ciunescu Emanuel
(L 1213-1217), Modora Vlad (L 1213-1217), B@na Mircea (L 1213-1217), Tulbure

Editura Cardinal 57
Andreea (L 1213-1217), clasa a X-a, prof. MIHAI DICU: C[lina Florian (L
1218-1222), clasa a XI-a, prof. ION P{TRA}CU: Betiu Pavel (L 1223-1227), clasa a
XI-a, prof. OANA PREDA: Etegan Valentina (L 1223-1227), Zanfir Bogdan (L
1223-1227), Duican Mihnea (L 1223-1227), C[pit[nescu Alexandra (L
1223-1227), S@rmon Georgiana (L 1223-1227), Lungu R[zvan (L 1223-1227),
Popescu Cristiana (L 1223-1227), clasa a XII-a, prof. LUCIAN TU|ESCU:
Nicol[escu Andrei (L 1228-1232), Sp[taru Andrei (L 1228-1232), Jianu Ligia (L
1228-1232). }coala Gim. Sf. Dumitru, clasa I, @nv. RALUCA DINU DIACONU:
Anastasescu Alexandru (I 1-25), L[p[datu Bianca (I 1-25), P[tru Grigorie (I
1-25), Bu\u }tefan (I 1-25).
CUGIR (jud. Alba): }coala Gim. Nr. 3, clasa a II-a, @nv. DORINA BR#NDU}E,
DORIN BR#NDU}E: Boruah Marasim (II 1-25), P[troescu M[d[lina (II 1-25),
Vulcu R[zvan (II 1-25), Cruceru Bianca (II 1-25), Pavel Andrei C[lin (II 1-25),
Suciu Alexia (II 1-25), Mih[ilescu Nicu]or (II 1-25), Olteanu Oana (II 1-25),
Nicoar[ Daria (II 1-25), Pascu Robert (II 1-25), Moise] Alexandru (II 1-25), clasa a
II-a, @nv. LAVINIA PUPU-FLE}ERIU: Ploia R[zvan (II 1-25), Mocan Nicoleta (II
1-25), Predescu Andreea (II 1-25), Suciu Giulia (II 1-25), Budere] Aurelia (II
1-25), Macarie Luana (II 1-25), Caraba] George (II 1-25), Mihai Cristian (II
1-25), F`naru Erika (II 1-25), Pascu Ionu\ (II 1-25), Balaciu Bianca (II 1-25),
Puican Georgiana (II 1-25), clasa a III-a, prof. @nv. primar ANDA TEC}A: Grecu
Iulian George (III 1-25), Mara Bianca (III 1-25), Micu Sebastian (III 1-25),
Molode\ Andrei Daniel (III 1-25), Zuga Daria Maria (III 1-25), clasa a III-a, prof.
@nv. primar SABINA PRESECAN ]i @nv. ELISABETA HADA: Ivini] Andrei (III
1-25), clasa a IV-a, @nv. LAVINIA LUPU-FLE}ERIU: Lupu-Fle]eriu Carol (IV
1-25).
LUPENI (jud. Hunedoara): Lic. Teoretic “Mircea Eliade”, clasa a VI-a, prof.
EMILIA VELCEA: Floronesc Riana Andreea (TG 4570, 4586, 4590), Paleac
Marina (TG 4570, 4580, 4583, 4586, 4590), Ninciuleanu Andreea (TG 4570, 4580,
4586, 4590), Csatlo] Andreea (TG 4586, 4590), B[loi Adrian (TG 4583, 4586,
4590), Some]an Mihai (TG 4580, 4583, 4586, 4590), Ilovan Adela (TG 4570, 4586,
4590), Fuieri Sava Ionel Darius (TG 4570, 4580, 4583), Iepure Ovidiu Bogdan
(TG 4570, 4580, 4583, 4586, 4590), clasa a VII-a, prof. EMILIA VELCEA: Topor
Vlad-Mihai (TG 4595, 4596, 4598, 4599), R[dulic[ Briana (TG 4595, 4596, 4598,
4599, 4600), Topliceanu Ariana Maria (TG 4595-4596, 4598, 4599), Suciu Ruben
(TG 4595, 4596, 4598), Fira Adela (TG 4595, 4596, 4598, 4599), Firescu Melisa

58 Revista de matematic[ din Craiova


(TG 4595, 4596, 4598, 4599), Beczuk Roberta (TG 4595, 4596, 4598, 4599), clasa a
VIII-a, prof. EMILIA VELCEA: Neam\u Laura (TG 4621, 4623, 4626, 4631, 4633),
Burger Andreea Ioana (TG 4621, 4623, 4626, 4631, 4633), Anghelu] Anca (TG
4623, 4626, 4631, 4633).
URECHE}TI (jud. Gorj): }coala Gim. Ureche]ti, clasa a III-a, prof. @nv. primar
VALENTINA-IRINA COVERC{: Balt[ Alexandru-Valentin (III 1-25), B[r[gan
Vasile-Alin (III 1-25), Eremia Irinel (III 1-25), Oproiu Emilia-Teodora (III 1-25),
Sanda Eugenia-Daniela (III 1-25), clasa a IV-a, prof. @nv. primar
VALENTINA-IRINA COVERC{: Bercan Alexandra-Raimonda (IV1-25), Bumbu
Constantin-Ciprian (IV1-25), C[ld[ru]e Ana-Maria (IV1-25), Macovei
Gabriel-Leonard (IV 1-25).
SATU-MARE (jud. Gorj): C.N. “Doamna Stanca”, clasa a X-a, prof. GIGEL
BUTH: Megye]an Alexandra (TL 3950; L 1220, 1221, 1218), Lup]e Vanessa (L
1220, 1221), Cauni Andra-Lea (TL 3950; L 1218, 1220, 1221, 1226), Medve David
(L 1220, 1221), Andreica Adrian-Marius (L 1220, 1221), Col\ A Tresilian (L 1220,
1221), Corodan David (L 1220, 1221).
TG. JIU (jud. Gorj): C.N. “Spiru Haret”, clasa a III-a, @nv. ILIE FLEANCU:
Popescu Maria (III 1-25).
TIMI}OARA (jud. Timi]): Lic. Pedagogic “Carmen Sylva”, clasa a II-a, prof.
@nv. primar SIMONA DELIA SAMFESCU: Barna Maria (II 1-12(c,d), 13-20,
22-25), B[loi Miriam (II 1-4, 8, 9, 11, 15-20, 22-25), Dumitrescu Anisia (II
1-12(c,d), 13-20, 22-25), Dungan Bogdan (II 1-12(c,d), 13-20, 22-25), Gabor
Andrei (II 1-12(c,d), 13-20, 22-25), G[lan Gabriel (II 1-12(c,d), 13-20, 22-25),
Lungoci Casian (II 1-12(c,d), 13-20, 24, 25), Lupa]cu Cezara (II 1-12(c,d), 13-20,
22-25), Mihu\[ Maria (II 1-12(c,d), 13-20, 22-25), Muia Patricia (II 1-12(c,d),
13-20, 22-25), N[chescu Andrada (II 1-12(c,d), 13-20, 22-25), Olariu Adriana (II
1-12(c,d), 13-20, 22-25), Pop Sara (II 1-12(c,d), 13-20, 22-25), R[ban Rebeca (II
1-12(c,d), 13-20, 22-25), Sfercoci R[zvan (II 1-12(c,d), 13-20, 22-25), Silea David
(II 1-12(c,d), 13-15, 18, 20, 22-25), S@rbu Bianca (II 1-12(c,d), 13-20, 22-25), }tefan
Andrei (II 1-12(c,d), 13-20, 22-25), Tal Karina (II 1-12(c,d), 13-20, 22-25), |urca]
Cristiana (II 1-12(c,d), 13-20, 22-25), Vidoni Ana Maria (II 1-12(c,d), 13-20,
22-25).
ZIMNICEA (jud. Teleorman): Lic. Teoretic Zimnicea, clasa a X-a, prof. TRAIAN
IANCULESCU: V[tafu Tiffany (TL 3950-3956, 3958, 3959, 3961, 3962, 3964,
3966, 3967, 3969, 3972, 3973, 3974), |ancu Mihaela-Isabelle (TL 3950-3954, 3956,

Editura Cardinal 59
3959, 3961, 3966, 3967, 3968, 3969, 3970, 3972), Laud[ Raluca (TL 3951-3953,
3956, 3959, 3966), Ghir\oiu Marian-Paul (TL 3951-3953, 3956, 3959, 3966),
Vizitiu Simona (TL 3951-3953, 3966), P[tra]cu Andreea (TL 3951-3953, 3956,
3959, 3966), Olan Alina (TL 3951-3953, 3956, 3959, 3961, 3966), Conceatu Silviu
(TL 3951-3953, 3956, 3959, 3966), Ciobanu Valeria (TL 3951-3953, 3956, 3959,
3966), P[tr[nescu Noela (TL 3952, 3953, 3956, 3959, 3961, 3966), Pu]ca] Denisa
(TL 3951-3953, 3956, 3959, 3961), |`n\arcu Alexandra (TL 3956, 3959, 3966),
Cr[ciunescu Delia (TL 3956, 3959, 3966).

*
* *

60 Revista de matematic[ din Craiova



NOTA REDAC|IEI: TALON DE PARTICIPARE
Solu\iile problemelor
propuse se vor trimite pe Nume..........................................Prenume..................................
adresa redac\iei @nso\ite
de talonul de participare Localitatea.........................................Jud...................................
completat p`n[ la data de }coala.........................................................Clasa........................
15.07.2017.
Profesor.......................................................................................


I.S.S.N. 1221- 3551

Lei 10

You might also like